PDA

نسخه کامل مشاهده نسخه کامل : اتاق ریاضیات(طرح سؤالات)



صفحه ها : 1 2 3 4 [5] 6 7 8 9 10 11 12 13 14 15 16 17 18 19 20

پاکر
15-10-2007, 20:19
am
خیلی ممنون از کمکتون ولی اون( 1- ) که گذاشتم به جای x هستش یعنی( f(x نه f وارون
دوست عزیز اگه تابع f در 1- تعریف شده باشه مقداری به شکل a که متعلق به R هست رو به دست میده
یعنی یک زوج مرتب به شکل (aو1-) میده که یک نقطه هستش
یعنی دامنه ی اوون 1- و بردش a خواهد بود


:11:

e_kh78
15-10-2007, 20:53
لطفا راهنمایی کنید جواب این چند مساله
1.حد X+1 بر روی X-1 کل عبارت کسری به توان 2X-1 وقتی X به سمت مثبت بینهایت میل میکند چه جوری میشه e به توان 4
2.حد X+1 بر روی X-2 کل مجموعه کسری به توان 2X-1 وقتی X به سمت مثبت بینهایت میل میکند چه جوری میشه e به توان 6

لطفا اگر ممکن است واضح توضیح دهید چون جواب را دارم ولی متوجه نمیشوم

خواهش میکنم این مساله هم جواب دهید

the dead
18-10-2007, 12:22
ميخواستم بدونم در محاسبه گشتاور چجوری ميشه تشخيص داد که ناحيه مسطح يا منحنی مسطح؟
مثلاً اين دو تابع در ناحيه مسطح هستش
[ برای مشاهده لینک ، لطفا با نام کاربری خود وارد شوید یا ثبت نام کنید ]
[ برای مشاهده لینک ، لطفا با نام کاربری خود وارد شوید یا ثبت نام کنید ]

مثلاً اين تابع در منحنی مسطح هستش
[ برای مشاهده لینک ، لطفا با نام کاربری خود وارد شوید یا ثبت نام کنید ]

mir@
18-10-2007, 19:59
لطفا راهنمایی کنید جواب این چند مساله
1.حد X+1 بر روی X-1 کل عبارت کسری به توان 2X-1 وقتی X به سمت مثبت بینهایت میل میکند چه جوری میشه e به توان 4
2.حد X+1 بر روی X-2 کل مجموعه کسری به توان 2X-1 وقتی X به سمت مثبت بینهایت میل میکند چه جوری میشه e به توان 6

لطفا اگر ممکن است واضح توضیح دهید چون جواب را دارم ولی متوجه نمیشوم


[ برای مشاهده لینک ، لطفا با نام کاربری خود وارد شوید یا ثبت نام کنید ] ft%28%5Cfrac%7Bx+1%7D%7Bx-1%7D%5Cright%29%5E%7B2x-1%7D

[ برای مشاهده لینک ، لطفا با نام کاربری خود وارد شوید یا ثبت نام کنید ] fty%7D%282x-1%29%5Cln%7B%5Cfrac%7Bx+1%7D%7Bx-1%7D%7D

[ برای مشاهده لینک ، لطفا با نام کاربری خود وارد شوید یا ثبت نام کنید ] fty%7D%5Cfrac%7B%5Cln%7B%5Cfrac%7Bx+1%7D%7Bx-1%7D%7D%7D%7B%5Cfrac%7B1%7D%7B2x-1%7D%7D

L'opital

[ برای مشاهده لینک ، لطفا با نام کاربری خود وارد شوید یا ثبت نام کنید ] fty%7D%5Cfrac%7B%5Cfrac%7B-2%7D%7Bx%5E2-1%7D%7D%7B%5Cfrac%7B-2%7D%7B%7B%282x-1%29%7D%5E2%7D%7D

[ برای مشاهده لینک ، لطفا با نام کاربری خود وارد شوید یا ثبت نام کنید ]

[ برای مشاهده لینک ، لطفا با نام کاربری خود وارد شوید یا ثبت نام کنید ]


اون يكي هم به اين شكل

aria_electric_fars
20-10-2007, 19:46
سلام دوستان
من می خوام یه 26 سوال انتگرال رو حل کنم فقط فوری تا امشب ساعت 10 بی زحمت کمکم کنید اگر برنامه ای هست که این کار رو می کنه و مثل حل به صورت دستی جواب می ده بهم بگید اگر هم خودتون فوری جواب می دید تا تند تند بزنم اگر هم راه دیگه ای هست بگید فقط فوری چون امشب آخرین وقتش هست

با تشکر
بای

aria_electric_fars
20-10-2007, 19:49
[ برای مشاهده لینک ، لطفا با نام کاربری خود وارد شوید یا ثبت نام کنید ]
[ برای مشاهده لینک ، لطفا با نام کاربری خود وارد شوید یا ثبت نام کنید ]
[ برای مشاهده لینک ، لطفا با نام کاربری خود وارد شوید یا ثبت نام کنید ]

[ برای مشاهده لینک ، لطفا با نام کاربری خود وارد شوید یا ثبت نام کنید ]
[ برای مشاهده لینک ، لطفا با نام کاربری خود وارد شوید یا ثبت نام کنید ]
[ برای مشاهده لینک ، لطفا با نام کاربری خود وارد شوید یا ثبت نام کنید ]

aria_electric_fars
20-10-2007, 19:59
[ برای مشاهده لینک ، لطفا با نام کاربری خود وارد شوید یا ثبت نام کنید ]
[ برای مشاهده لینک ، لطفا با نام کاربری خود وارد شوید یا ثبت نام کنید ]
[ برای مشاهده لینک ، لطفا با نام کاربری خود وارد شوید یا ثبت نام کنید ]
[ برای مشاهده لینک ، لطفا با نام کاربری خود وارد شوید یا ثبت نام کنید ]
[ برای مشاهده لینک ، لطفا با نام کاربری خود وارد شوید یا ثبت نام کنید ]
[ برای مشاهده لینک ، لطفا با نام کاربری خود وارد شوید یا ثبت نام کنید ]

aria_electric_fars
20-10-2007, 20:12
[ برای مشاهده لینک ، لطفا با نام کاربری خود وارد شوید یا ثبت نام کنید ]
[ برای مشاهده لینک ، لطفا با نام کاربری خود وارد شوید یا ثبت نام کنید ]

[ برای مشاهده لینک ، لطفا با نام کاربری خود وارد شوید یا ثبت نام کنید ]
[ برای مشاهده لینک ، لطفا با نام کاربری خود وارد شوید یا ثبت نام کنید ]
[ برای مشاهده لینک ، لطفا با نام کاربری خود وارد شوید یا ثبت نام کنید ]
[ برای مشاهده لینک ، لطفا با نام کاربری خود وارد شوید یا ثبت نام کنید ]

aria_electric_fars
20-10-2007, 20:15
دوستان
فعلا اینا رو جواب بدید اگر اینا رو جواب دادید ما بقی رو فوری آپلود می کنم بی زحمت همین امشب من همین امشب رو وقت دارم اگر جواب ندادید تا امشب دیگه نمی خواد جوابش رو بدید

با تشکر
بای

Hamid69
21-10-2007, 19:43
سلام.
کسی جواب این سوال رو بلده؟البته تا سه شنبه لازمش دارم.ممنون.
سوال:

در زمینی مستطیل شکل با ابعاد 72 و 114 متر بوته هایی به فواصل حداقل 3 متر از یکدیگر کاشته ایم. حداکثر چند بوته کاشته ایم؟

mashaheeer
23-10-2007, 14:33
با سلام
يه مقدار اطلاعات درباره نمونه تصادفي ميخواهم
لطفا اگه اطلاعاتي داريد هرچه زودتر بنويسيد.فردا بايد كنفرانس بدم.

Dikenz
24-10-2007, 09:41
سلام.
کسی جواب این سوال رو بلده؟البته تا سه شنبه لازمش دارم.ممنون.
سوال:

در زمینی مستطیل شکل با ابعاد 72 و 114 متر بوته هایی به فواصل حداقل 3 متر از یکدیگر کاشته ایم. حداکثر چند بوته کاشته ایم؟

اگر منظورت اين است که بوته ها با هم از هر طرف 3 متر فاصله دارند خوب طبيعتا 912بوته خواهيم داشت.

Dikenz
24-10-2007, 11:52
با عرض سلام به همه ی ریاضی دوستان عزیز

به خصوص آقای مفیدی

من هم ورودم را با ساده ترین سوال سخت شروع میکنم

ثابت کنید هرعدد حقیقی غیر صفر ، به توان صفر برابر یک می باشد

a^0 = a^m/a^m = 1

Dikenz
24-10-2007, 11:53
راستي سلام
ببخشد که پابرهنه وارد شدم
خوشحال ميشم که من را در جمع خود بپذيريد

Dikenz
24-10-2007, 11:54
اگه بخواد دو یا بیش از یک مجموعه مرجع وجود داشته باشه یا با هم مساوی هستند که در این صورت متمایز نیستند و در این صورت چند مجموعه نبوده و یه مجموعه بوده. اگه بخواند مساوی نباشند، عضوی توی یکی هست که تو دیگری نیست، پس یکی‌شون مرجع نیست چون تمام اعداد و مجموعه‌ها رو در بر نگرفته. پس مجموعه مرجه یکتاست.

البته توجه داريد که دسته اي از تمام مجموعه ها خود، مجموعه نيست

zibaandish
24-10-2007, 16:23
من در حل مساله اي دچار مشكل هستم ميتونيد كمكم كنيد؟اينه:
اگر pعددي اول باشد جوابهاي صحيح معادله زير را بيابيد:
p(x+y)=xy

farbod123
25-10-2007, 15:50
سلام.:40:
من يك سؤال دارم كه شايد به نظر خيلي از شما سطحش پايين باشه ولي از شما خواهش مي كنم جواب بديد...خيلي حياتيه...:41:
روي سوال لين طوريه:
در يك جعبه مداد رنگي 7مداد رنگي وجود دارد.به چند طريق مي توان مداد ها را داخل جعبه چيد؟(به طوري كه فاصلهء بين مداد ابي و قرمز يك مداد باشه):41::11:
جواب رو ميتونيد به شكل فاكتور يل(!) هم بنويسيد...:19:
ضمناَ راه حلتون رو هم بنويسيد...:19:

SuB
26-10-2007, 14:10
البته توجه داريد که دسته اي از تمام مجموعه ها خود، مجموعه نيست

والا تا اونجایی که من بلدم واژه‌های مجموعه، دسته و ... همگی یکی هستند و در واقع بیانگر همون مجموعه‌اند. پس دسته یا مجموعه‌ای از مجموعه‌ها، خود یک مجموعه است. مثلاً مجموعه توانی خودش مجموعه‌ای از تمام زیرمجموعه‌های یک مجموعه هست!!!:21:

SuB
26-10-2007, 14:19
سلام.:40:
من يك سؤال دارم كه شايد به نظر خيلي از شما سطحش پايين باشه ولي از شما خواهش مي كنم جواب بديد...خيلي حياتيه...:41:
روي سوال لين طوريه:
در يك جعبه مداد رنگي 7مداد رنگي وجود دارد.به چند طريق مي توان مداد ها را داخل جعبه چيد؟(به طوري كه فاصلهء بين مداد ابي و قرمز يك مداد باشه):41::11:
جواب رو ميتونيد به شكل فاكتور يل(!) هم بنويسيد...:19:
ضمناَ راه حلتون رو هم بنويسيد...:19:

فکر کنم بشه 10*!5

farbod123
26-10-2007, 15:18
فکر کنم بشه 10*!5
ميشه راه حلتون رو بنويسيد؟؟؟:11:
خيلي ممنون...:40:

پاکر
26-10-2007, 16:02
والا تا اونجایی که من بلدم واژه‌های مجموعه، دسته و ... همگی یکی هستند و در واقع بیانگر همون مجموعه‌اند. پس دسته یا مجموعه‌ای از مجموعه‌ها، خود یک مجموعه است. مثلاً مجموعه توانی خودش مجموعه‌ای از تمام زیرمجموعه‌های یک مجموعه هست!!!:21:
بله دقیقا طبق مفهموم مجموعه اجتماع چند شیء یا هر چیز دیگری یک مجموعه به حساب می آید:11:

SuB
26-10-2007, 23:38
ميشه راه حلتون رو بنويسيد؟؟؟:11:
خيلي ممنون...:40:

محل قرارگیری مدادها رو توی جامدادی، 7 خونه در نظر گرفتم.
گفتم باید بین آبی و قرمز یه مداد فاصله باشه. اول فرض کردم اولین خونه قرمز و سومین خونه آبی باشه. پس 5 تا خونه باقی‌ می‌مونه با 5 تا مداد که میشه !5 حالت. حالا این مدادهای آبی و قرمز می‌تونند به 5 روش با ترتیب اول قرمز و بعد یه رنگ دیگه و بعد آبی قرار بگیرند. 5 حالت هم اول آبی بعد یه رنگ دیگه و بعد قرمز. جمعاً میشه 10 حالت و طبق اصل شمارش میشه همون که گفتم. یعنی 10*!5
:11:

glileh
27-10-2007, 19:01
ببخشید تا می تونید سریع جواب این سول رو بدین ! 0 به قوه صفر چی میشه ! لطفا ً کامل ، واضح و ساده ترین جواب رو بدین ! ممکنه سطحش پایین باشه ولی .... لطفا پاسخ بدین ! ( با راه حل )

mir@
27-10-2007, 19:44
صفر فاكتوريل كه ميشه 1

هر عدد هم به توان صفر ميشه 1

پس كلاَ ميشه يك

پاکر
27-10-2007, 21:12
ببخشید تا می تونید سریع جواب این سول رو بدین ! 0 به قوه صفر چی میشه ! لطفا ً کامل ، واضح و ساده ترین جواب رو بدین ! ممکنه سطحش پایین باشه ولی .... لطفا پاسخ بدین ! ( با راه حل )


صفر فاكتوريل كه ميشه 1

هر عدد هم به توان صفر ميشه 1

پس كلاَ ميشه يك
البته جسارت نباشه آقا mir@ من حدس میزنم منظور ایشون شاید این باشه
0^0 که مبهم میشه
اگه منظورشون این نبوده شرمنده:11:

golabi2
28-10-2007, 11:36
به نام خدا
2تا سوال دارم تو يه فايل notepad آپلود کردم (با تايپش مشکل داشتم)
سوال اول مجموعه ي تواني خواسته
دومي بايد اثبات بشه راستي فردا بايد تحويل بدم اگه ميشه يکم سريع:blush:
[ برای مشاهده لینک ، لطفا با نام کاربری خود وارد شوید یا ثبت نام کنید ] لينک دانلود سوالها

glileh
28-10-2007, 17:05
البته جسارت نباشه آقا mir@ من حدس میزنم منظور ایشون شاید این باشه
0^0 که مبهم میشه
اگه منظورشون این نبوده شرمنده:11:





چرا منظور من دقیقا ً همین بوده . ازتون ممنونم . دیگه دیر شد . باید امروز می دادم . ولی اگر هم این طور نبود " مبهم می شه " جواب نبود . باید خیلی تو ضیح بیشتری می دادم .:40:

پاکر
28-10-2007, 19:33
چرا منظور من دقیقا ً همین بوده . ازتون ممنونم . دیگه دیر شد . باید امروز می دادم . ولی اگر هم این طور نبود " مبهم می شه " جواب نبود . باید خیلی تو ضیح بیشتری می دادم .:40:
بهتره اینجور اصلاح کنم که میشه تعریف نشده! حالا من که دقیقا نفهمیدم منظور شما چی بود!:11:

golabi2
29-10-2007, 04:16
ممنون از جوابتون

SuB
29-10-2007, 21:19
چرا منظور من دقیقا ً همین بوده . ازتون ممنونم . دیگه دیر شد . باید امروز می دادم . ولی اگر هم این طور نبود " مبهم می شه " جواب نبود . باید خیلی تو ضیح بیشتری می دادم .:40:

1- هر عدد غیر صفر به توان سفر میشه 1 نه هر عدد.:46:
2- مشکل اینجاست که شما حساب رو با حد گیری غاطی می‌کنید.:13: توی حساب صفر تقسیم بر صفر معنی نداره ولی توی حد گیری صفر روی صفر یک حالت مبهم محسوب میشه. رسیدن به حالت صفر صفرم به حاطر این هست که شما می‌خواهید بدون در نظر گرفتن شرایط قضیه‌های حد گیری، حد بگیرید. برای همین بهش میگند مبهم:11:

توی حساب صفر به توان صفر معنی نداره. نه مبهم میشه نه هیچ چیز دیگه. ولی توی حدگیری صفر به توان صفر حالت مبهمی است که باید رفع ابهام بشه.

وقتی میگی قوه یادم به دبیر شیمی‌مون می‌افته که یه 60 70 سالی سن داره و به جای به توان،میگه به قوه!:27:

hitman45
29-10-2007, 23:25
با سلامبراي حل مسئله زير راه حلتون چيه؟[ برای مشاهده لینک ، لطفا با نام کاربری خود وارد شوید یا ثبت نام کنید ]

Dikenz
30-10-2007, 07:55
والا تا اونجایی که من بلدم واژه‌های مجموعه، دسته و ... همگی یکی هستند و در واقع بیانگر همون مجموعه‌اند. پس دسته یا مجموعه‌ای از مجموعه‌ها، خود یک مجموعه است. مثلاً مجموعه توانی خودش مجموعه‌ای از تمام زیرمجموعه‌های یک مجموعه هست!!!:21:
فکر کنم در کتاب مباني رياضيات ابراهيمي آمده است که : طبق اصل منظم بودن مجموعه ها هيچ مجموعه اي عضو خودش نيست پس اگر دسته اي که تمام مجموعه ها را در بردارد خود مجوعه باشد بايد عصو خودش باشد.

Dikenz
30-10-2007, 08:02
بله دقیقا طبق مفهموم مجموعه اجتماع چند شیء یا هر چیز دیگری یک مجموعه به حساب می آید:11:
فکر ميکنيد اين مجموعه است: {1و1و1و1}؟

Dikenz
30-10-2007, 08:19
با سلامبراي حل مسئله زير راه حلتون چيه؟[ برای مشاهده لینک ، لطفا با نام کاربری خود وارد شوید یا ثبت نام کنید ]

منظورت اينه که ردش کنيم؟ پس این هم مثال نقض: براي n=2

3/1+1/1>2/2

SuB
30-10-2007, 22:33
با سلامبراي حل مسئله زير راه حلتون چيه؟[ برای مشاهده لینک ، لطفا با نام کاربری خود وارد شوید یا ثبت نام کنید ]

من شک دارم این مسئله جبر و احتمال سال سوم دبیرستان باشه. چون گفته برای هر عدد حقیقی r و عدد حقیقی به توان عدد گنگ رو توی دبیرستان اصلاً نمی‌خونید.:10:

SuB
30-10-2007, 22:39
فکر کنم در کتاب مباني رياضيات ابراهيمي آمده است که : طبق اصل منظم بودن مجموعه ها هيچ مجموعه اي عضو خودش نيست پس اگر دسته اي که تمام مجموعه ها را در بردارد خود مجوعه باشد بايد عصو خودش باشد.

من این کتاب رو نخوندم. تا حالا اصل منظم بودن مجموعه‌ها رو هم نشنیدم. میشه بگید چی هست؟

در ضمن {1و1و1و1} مجموعه هست و همون {1} می‌باشد.:46:

Pouriaqazvin
30-10-2007, 22:43
فکر ميکنيد اين مجموعه است: {1و1و1و1}؟
خب من سال دوم دبیرستان هستم ولی میدونم این یک مجموعه است و و عدد یک تنها در اینجا تکرار شده و مجموعه برابر {1} است

پاکر
30-10-2007, 23:04
فکر ميکنيد اين مجموعه است: {1و1و1و1}؟
بدیهیه که این یه مجموعه ی تک عضوی هستش... خب یه بارم گفتم اجتماع هر شیء یا هر چیز دیگه تشکیل یه مجموعه میده:11:

منظورت اينه که ردش کنيم؟ پس این هم مثال نقض: براي n=2


3/1+1/1>2/2

منظور ایشون استقرای تعمیم یافته هستش که به ازای یکسری n قابل قبول میشه منظور از عدد حقیقی هم همون کسر ها هستش و ربطی به n نداره:11:

mahdi bg
31-10-2007, 05:45
سلام
جواب این دوتا سیکما چی میشه

[ برای مشاهده لینک ، لطفا با نام کاربری خود وارد شوید یا ثبت نام کنید ]

[ برای مشاهده لینک ، لطفا با نام کاربری خود وارد شوید یا ثبت نام کنید ]

لطفا راه حل رو هم بذارین
ممنون

hitman45
31-10-2007, 07:40
منظورت اينه که ردش کنيم؟ پس این هم مثال نقض: براي n=2


3/1+1/1>2/2



من شک دارم این مسئله جبر و احتمال سال سوم دبیرستان باشه. چون گفته برای هر عدد حقیقی r و عدد حقیقی به توان عدد گنگ رو توی دبیرستان اصلاً نمی‌خونید.:10:
با سلام
البته منظور حل با استفاده از استقراي تعميم يافته هستش كه به ازاي n=3 به بعد اثبات بشه يعني اول پايه استقرا رو پيدا كنيم كه تعيين شد سپس با فرض گرفتن p(n انگاه p(n+1 را اثبات كنيم

pp8khat
31-10-2007, 14:48
تو این تعمیم یافته نمی شه از صفر یا منفی 1 شروع کرد؟

پاکر
31-10-2007, 15:05
تو این تعمیم یافته نمی شه از صفر یا منفی 1 شروع کرد؟
چرا نمیشه عزیزم شما یکم فکر کنی می بینی که اسم این هستش استقرای تعمیم یافته در حالی که توی سوم شما بدتر مقادیر n رو محدودتر میکنین، ایشالا توی پیش دانشگاهی با اصلش آشنا میشی:11:

Dikenz
31-10-2007, 18:01
خب من سال دوم دبیرستان هستم ولی میدونم این یک مجموعه است و و عدد یک تنها در اینجا تکرار شده و مجموعه برابر {1} است

فکر کنم در رياضي سال اول دبيرستان امده است که مجموعه عبارتست از دسته اي از اشياء مشخص و دو به دو متمايز

Dikenz
31-10-2007, 18:02
[quote=پاکر;1766083]بدیهیه که این یه مجموعه ی تک عضوی هستش... خب یه بارم گفتم اجتماع هر شیء یا هر چیز دیگه تشکیل یه مجموعه میده:11:

بديهيه که اين يک خانواده است و نه مجوعه. (به همان منبع مراجعه کنيد.)

Dikenz
01-11-2007, 11:00
من این کتاب رو نخوندم. تا حالا اصل منظم بودن مجموعه‌ها رو هم نشنیدم. میشه بگید چی هست؟

در ضمن {1و1و1و1} مجموعه هست و همون {1} می‌باشد.:46:

والله من هم همين مقدار و فقظ در همين کتاب خوانده ام. درضمن منظور من اصلا اين نبود که مجموعه مجموعه ها نداريم (که شما مجموعه تواني را مثال زديد) بلکه منظورم اين بود که مجموعه اي شامل تمام مجموعه ها نداريم.

Dikenz
01-11-2007, 11:02
با سلام
البته منظور حل با استفاده از استقراي تعميم يافته هستش كه به ازاي n=3 به بعد اثبات بشه يعني اول پايه استقرا رو پيدا كنيم كه تعيين شد سپس با فرض گرفتن p(n انگاه p(n+1 را اثبات كنيم

ببخشيد. ولي باور کنيد که سوالتان مبهم بود.

interior_design
01-11-2007, 11:05
دوستان عزیز من توضبح 2 روش دو بخشی و نابجایی رو میخوام با مقایسه این دو.

ممنون

tellme
01-11-2007, 14:25
این سوال رو لطفا برام حل کنید خیلی عجله دارم واسه پیدا کردن این جواب :

در یک مسابقه دو (ورزش) به طول X و با سرعت ثابت , A از B به اندازه 20متر , B از C به اندازه 10متر و A از C به اندازه 28متر پیشی میگیرند . X برابر است با چه عددی؟؟ :13:

لطفا این رو واسم حل کنید!/..

pp8khat
01-11-2007, 17:19
بیشتر به سوال فیزیک می خوره...
تا 10 دقیقه دیگه حلش می کنم(کافیه یه x'ox بکشی...)
آخر مسابقه چی میشه؟؟
یعنی کی اول می شه کی دوم؟؟

Vahid67
01-11-2007, 18:18
سلام
اگه میشه یه کمکی بکنید این چندتا انتگرال حل بشه
[ برای مشاهده لینک ، لطفا با نام کاربری خود وارد شوید یا ثبت نام کنید ]

Pouriaqazvin
01-11-2007, 20:37
فکر کنم در رياضي سال اول دبيرستان امده است که مجموعه عبارتست از دسته اي از اشياء مشخص و دو به دو متمايز
من از نوع تفکر شما اطلاع ندارم ولی ما {} رو هم یک مجموعه می گیریم باز که نمی خوای بگی این اعضاش دو به دو متمایز نیستن؟؟؟؟؟؟؟؟؟؟!!!!!!!!!!!!

mahdi bg
02-11-2007, 05:17
سلام
کسی خواص و ویژگی های سیکما رو می دونه؟
ممنون

SuB
02-11-2007, 10:47
فکر کنم در رياضي سال اول دبيرستان امده است که مجموعه عبارتست از دسته اي از اشياء مشخص و دو به دو متمايز
این کاری که شما دارید میکنید ارئه تعریف برای مجموعه هست که مجموعه تعریف نداره. در ضمن شما از عبارت دسته استفاده کرده‌اید که باید خود دسته رو هم تعریف کنید.


بديهيه که اين يک خانواده است و نه مجوعه. (به همان منبع مراجعه کنيد.)
حالا خانواده چی هست؟:27:

احتمالاً اون منبع شما می‌خواد مثل کسانی باشه که هندسه نااقلیدسی رو بنا کردند!:46:

tellme
02-11-2007, 11:54
pp8khat : من که نگفتم مسابقه هستش :D

فقط خواستم از شما یا هر کس دیگه که برام به طور صحیح حل کنه چون هر کار کردم نتونستم یکی واسم حل کرد ولی میگه خودت باز برو دنبالش

پاکر
02-11-2007, 19:27
با سلامبراي حل مسئله زير راه حلتون چيه؟[ برای مشاهده لینک ، لطفا با نام کاربری خود وارد شوید یا ثبت نام کنید ]
از این خلاصه ترم میشد اما گفتم کامل باشه:11:
[ برای مشاهده لینک ، لطفا با نام کاربری خود وارد شوید یا ثبت نام کنید ]

hitman45
02-11-2007, 20:08
از این خلاصه ترم میشد اما گفتم کامل باشه:11:
[ برای مشاهده لینک ، لطفا با نام کاربری خود وارد شوید یا ثبت نام کنید ]

با سلام
از بابت راه حلتون ممنون متشكر [ برای مشاهده لینک ، لطفا با نام کاربری خود وارد شوید یا ثبت نام کنید ]

پاکر
03-11-2007, 01:30
با سلام
از بابت راه حلتون ممنون متشكر [ برای مشاهده لینک ، لطفا با نام کاربری خود وارد شوید یا ثبت نام کنید ]
یه اشکال کوچیکی بود رفع شد:11:

hitman45
03-11-2007, 07:15
یه اشکال کوچیکی بود رفع شد:11:
با سلام و تشكر از بابت حل مسئله

popeye
04-11-2007, 16:48
ايناست:

x-x^2/2<ln(1+x)<x x>0 - 1

2-tanx>x+x^3/3 بين xصفر تا پي دوم


x/x+1<ln(1+x)<x x>0 -3

يه ( jentel(man or woman
جواب اين سوالارو بده

پاکر
04-11-2007, 17:54
دوست عزیز لطفا سوال خود را در اتاق طرح سوالات ریاضی مطرح کنید:11:

Dikenz
05-11-2007, 13:22
من از نوع تفکر شما اطلاع ندارم ولی ما {} رو هم یک مجموعه می گیریم باز که نمی خوای بگی این اعضاش دو به دو متمایز نیستن؟؟؟؟؟؟؟؟؟؟!!!!!!!!!!!!

اولا که تهي دسته اي از 0 شي مشخص و دوبدو متمايز است و دوما در آن هيچ عضوي يافت نمي شود که متمايز نباشد. ضمنا بهتره اين بحث را تمام کنيم و به مسائل مفيدتري بپردازيم.

metalgirl
05-11-2007, 16:02
اگر pاندیس n , nامین عدد اول باشد دنبالهp اندیس nبه توان 1 n ام چیست؟؟؟؟؟؟؟؟؟؟؟؟؟؟؟؟؟؟؟؟؟

metalgirl
05-11-2007, 16:20
اگرpاندیسn,nامین عدد اول باشد دنبالهpاندیسnبه توان1 n ام چیست؟؟؟؟؟؟؟؟؟؟/

SuB
05-11-2007, 22:27
سلام
منظور از مجموعه متناهی چیست؟
مجموعه‌ای که کراندار باشد یا مجموعه‌ای که تعداد اعضای آن قابل شمارش باشند؟

ali1234
06-11-2007, 09:45
با سلام
يه سوال داشتم :
ديروز هوا o درجه بود . امروز هوا دو برابر سرد تر شده است و مردم لباس هاي بيشتري پو شيده اند . امروز هوا چند درجه است ؟

mir@
06-11-2007, 10:02
با سلام
يه سوال داشتم :
ديروز هوا o درجه بود . امروز هوا دو برابر سرد تر شده است و مردم لباس هاي بيشتري پو شيده اند . امروز هوا چند درجه است ؟

چون اين درجه به سانتي گراده .... پس به كلوين ميشه 273 درجه

حالا اگر بخواد دوبرابر سرد تر بشه ... احتمالاً بايد دما نصف بشه .. يعني ميشه فكر كنم 186.5 درجه كلوين.

يعني 186.5 درجه سانتي‌گراد زير صفر [ برای مشاهده لینک ، لطفا با نام کاربری خود وارد شوید یا ثبت نام کنید ]

amir_a
06-11-2007, 18:29
سلام.

کسی میدونه با کاسیو الجبرا چه جوری میشه تجزیه کسر انجام داد؟


:41:

Dikenz
08-11-2007, 09:45
سلام
منظور از مجموعه متناهی چیست؟
مجموعه‌ای که کراندار باشد یا مجموعه‌ای که تعداد اعضای آن قابل شمارش باشند؟

با سلام.
طبعا نمی توان مجموعه کراندار را متناهی دانست زيرا بعنوان مثال هر بازه ای کراندار است اما متناهی نیست.
و منظور از قابل شمارش را هم نمی دانم اما این را می دانم که مجموعه ای متناهی است که با n اندیس k مفروضی یعنی اعداد طبیعی تا k یکریخت باشد.

Dikenz
08-11-2007, 10:09
اگر pاندیس n , nامین عدد اول باشد دنبالهp اندیس nبه توان 1 n ام چیست؟؟؟؟؟؟؟؟؟؟؟؟؟؟؟؟؟؟؟؟؟

جالبه. میشه اعداد اصم.

marmdn
08-11-2007, 21:08
خب شايد جاش اين جا نباشه و خيلي راحت باشه ولي پاسخ سوال پايين رو لطفا با فرمول يا راه حل نياز دارم:
يك بيست وجهي يك چند وجهي منتظم با بيست وجه به شكل مثلث مي باشد.اين شكل چند يال دارد؟

SuB
09-11-2007, 09:46
با سلام.
طبعا نمی توان مجموعه کراندار را متناهی دانست زيرا بعنوان مثال هر بازه ای کراندار است اما متناهی نیست.
و منظور از قابل شمارش را هم نمی دانم اما این را می دانم که مجموعه ای متناهی است که با n اندیس k مفروضی یعنی اعداد طبیعی تا k یکریخت باشد.

شما که بدترش کردید.
یکریخت دیگه چیه؟
من که منظور این رو نمی‌فهمم:مجموعه ای متناهی است که با n اندیس k مفروضی یعنی اعداد طبیعی تا k یکریخت باشد.

alishahbazi
09-11-2007, 10:56
با سلام

فرمول اول میانگین، دومی واریانس و سومی انحراف معیار n تا متغیر تصادفی است:


[ برای مشاهده لینک ، لطفا با نام کاربری خود وارد شوید یا ثبت نام کنید ]

موفق باشید.


سلام خدمت تمامی دوستان

دوست عزیز mofidy1 جان اگه میشه یک مثال بزنی که من متوجه بشم خیلی ممنونت میشم.
با تشکرات فراوان(علی شهبازی)

the dead
14-11-2007, 08:31
[ برای مشاهده لینک ، لطفا با نام کاربری خود وارد شوید یا ثبت نام کنید ]
[ برای مشاهده لینک ، لطفا با نام کاربری خود وارد شوید یا ثبت نام کنید ]
[ برای مشاهده لینک ، لطفا با نام کاربری خود وارد شوید یا ثبت نام کنید ]

Dikenz
14-11-2007, 11:35
شما که بدترش کردید.
یکریخت دیگه چیه؟
من که منظور این رو نمی‌فهمم:مجموعه ای متناهی است که با n اندیس k مفروضی یعنی اعداد طبیعی تا k یکریخت باشد.

بسيار خوب. دو مجموعه را يکريخت گويند هرگاه بين آنها تابعي دو سويه وجود داشته باشد
و اما درباره قسمت دوم: مثلا N انديس 5 برابر است با 1و2و3و4
درمجموع مجموعه A را متناهي گويند اگر به ازاي هر عضو آن يک عدد طبيعي را متناظر کنيم که فکر کنم بهتر است همان قابل شمارش بودن را بگوييم. حالا مي شود شما قابل شمارش بودن را تعريف کند؟

the dead
15-11-2007, 13:23
[ برای مشاهده لینک ، لطفا با نام کاربری خود وارد شوید یا ثبت نام کنید ]
[ برای مشاهده لینک ، لطفا با نام کاربری خود وارد شوید یا ثبت نام کنید ]
[ برای مشاهده لینک ، لطفا با نام کاربری خود وارد شوید یا ثبت نام کنید ]

تو رو خدا يکی جواب بده !!!
سوال سومی رو تو اون جايی که خط کشيدم(در جوابش) نفهميدم که چه کار کرده

mir@
15-11-2007, 14:34
[ برای مشاهده لینک ، لطفا با نام کاربری خود وارد شوید یا ثبت نام کنید ]
[ برای مشاهده لینک ، لطفا با نام کاربری خود وارد شوید یا ثبت نام کنید ]
[ برای مشاهده لینک ، لطفا با نام کاربری خود وارد شوید یا ثبت نام کنید ]

واضحه كه ...

نوشته

[ برای مشاهده لینک ، لطفا با نام کاربری خود وارد شوید یا ثبت نام کنید ]

حالا از دوطرف ديفرانسيل بگير (يعني همون مشتق به اضافه نمو هر متغير)

[ برای مشاهده لینک ، لطفا با نام کاربری خود وارد شوید یا ثبت نام کنید ]

حالا انتگرال رو به اين صورت بنويس:

[ برای مشاهده لینک ، لطفا با نام کاربری خود وارد شوید یا ثبت نام کنید ] %28x%29%29%281+%5Ctan%5E2%28x%29%29dx

حالا معلومه كه[ برای مشاهده لینک ، لطفا با نام کاربری خود وارد شوید یا ثبت نام کنید ] لذا با رساندن طرفين به توان 2 ميرسيم به [ برای مشاهده لینک ، لطفا با نام کاربری خود وارد شوید یا ثبت نام کنید ] و[ برای مشاهده لینک ، لطفا با نام کاربری خود وارد شوید یا ثبت نام کنید ] 9 و همچنين طبق مشتقي كه گرفتيم [ برای مشاهده لینک ، لطفا با نام کاربری خود وارد شوید یا ثبت نام کنید ]

بنابراين انتگرالي كه در بالا نوشتم ميشه [ برای مشاهده لینک ، لطفا با نام کاربری خود وارد شوید یا ثبت نام کنید ]

اميدوارم مشخص باشه

pp8khat
15-11-2007, 16:19
خواهشاً یکی به این سوال هندسه جواب بده!!
* مجموع تعداد اقطار یک چهار ضلعی محدب با یک چهار ضلعی غیر محدب چند است؟
1-1
2-2
3-3
4-4
ممنون

SuB
15-11-2007, 22:14
بسيار خوب. دو مجموعه را يکريخت گويند هرگاه بين آنها تابعي دو سويه وجود داشته باشد
درمجموع مجموعه A را متناهي گويند اگر به ازاي هر عضو آن يک عدد طبيعي را متناظر کنيم که فکر کنم بهتر است همان قابل شمارش بودن را بگوييم. حالا مي شود شما قابل شمارش بودن را تعريف کند؟

شما می‌خواهید بهترش کنید، بدترش میکنید.

حالا باید تابع دو سویه رو تعریف کنید!:31:



درمجموع مجموعه A را متناهي گويند اگر به ازاي هر عضو آن يک عدد طبيعي را متناظر کنيم که فکر کنم بهتر است همان قابل شمارش بودن را بگوييم. حالا مي شود شما قابل شمارش بودن را تعريف کند؟
:18:
مجموعه اعداد طبیعی رو در نظر بگیرید. واضحه که به ازای هر عضو مجموعه اعداد طبیعی، یک عدد طبیعی متناظر می‌شود. ولی همه ما می‌دانیم که مجموعه اعداد طبیعی نامتناهی است!:21:
همچنین اند مجموعه اعداد زوج و مجموعه اعداد فرد و مجموعه اعداد صحیح و ....

اینم یه تعریف از خودم:
اعضای یک مجموعه را قابل شمارش گویند هر گاه تعداد اعضای آن مجموعه، یک عدد حسابی (اشتراک صفر و مجموعه اعداد طبیعی را مجموعه اعداد حسابی گویند و هر عضو از این مجموعه را یک عدد حسابی گویند) معین باشد.
:11:

پاکر
16-11-2007, 07:37
تو رو خدا يکی جواب بده !!!
سوال سومی رو تو اون جايی که خط کشيدم(در جوابش) نفهميدم که چه کار کرده
[ برای مشاهده لینک ، لطفا با نام کاربری خود وارد شوید یا ثبت نام کنید ]
توجه کنین که این انتگرال نامعین هستش نه مقدار:11:

sd70
16-11-2007, 10:07
عزيزان من چند سوال هندسه داشتم. به جواب شون هم احتياج دارم. لطفا کمک کنيد. (در حد سوم دبيرستان)

1- نيمساز B از مثلث ABC را رسم مي کنيم تا ضلع AC را در نقطه اي چون M قطع کند. MA را بر حسب طول اضلاع بدست آوريد.

2- نشان دهيد اگر نيمساز خارجي يک زاويه مثلثي با ضلع روبروي آن زاويه موازي باشد متساوي الساقين است.

3- مثلي زاويه 150 درجه دارد و طول اضلاع اين زاويه a و b است. مساحت آن را بيابيد.

4- زاويه ارتفاع وارد بر وتر در يک ضلع زاويه قائمه دقيقا برابر زاويه ميانه وارد بر وتر با ضلع ديگر زاويه قائمه است.

5- در هر مثلث قائم الزاويه مربع هر ضلع زاويه قائمه برابر است با حاصلضرب وتر در تصوير آن ضلع بر وتر.

6- در هر مثلث قائم الزاويه ارتفاع وارد بر وتر، واسطه هندسي بين دو قطعه است که ارتفاع روي وتر را ايجاد مي کند.

7- از تمام مثلث هايي که دو ضلع آنها ثابت باشد مساحت کدام يک ماکسيمم است؟

8- در يک چهارضلعي دو قطر زاويه 30 درجه با هم تشکيل داده اند. مساحت چهارضلعي را بر حسب دو قطر بنويسيد.

9- نشان دهيد هر گاه زواياي مجاور به هر ضلع در چهارضلعي مکمل باشند آن چهار ضلعي متوازي الاضلاع است.

Dikenz
17-11-2007, 08:19
:18:
مجموعه اعداد طبیعی رو در نظر بگیرید. واضحه که به ازای هر عضو مجموعه اعداد طبیعی، یک عدد طبیعی متناظر می‌شود. ولی همه ما می‌دانیم که مجموعه اعداد طبیعی نامتناهی است!:21:
همچنین اند مجموعه اعداد زوج و مجموعه اعداد فرد و مجموعه اعداد صحیح و ....

اینم یه تعریف از خودم:
اعضای یک مجموعه را قابل شمارش گویند هر گاه تعداد اعضای آن مجموعه، یک عدد حسابی (اشتراک صفر و مجموعه اعداد طبیعی را مجموعه اعداد حسابی گویند و هر عضو از این مجموعه را یک عدد حسابی گویند) معین باشد.
:11:
[/QUOTE]
:31:درواقع من آمدم به زبان ساده تر توضیح بدم ولی بدتر شد ظاهرا همان متناظر بودن با اعداد طبیعی تا k بهتر است. تعریف شما هم خوب است اما با محدودیت هایی مواجه است مثلا آیا می توانید بگویید این مجوعه متناهی است یا نه: { a,g,h,5,6,...,b}؟

SuB
17-11-2007, 12:52
:31:درواقع من آمدم به زبان ساده تر توضیح بدم ولی بدتر شد ظاهرا همان متناظر بودن با اعداد طبیعی تا k بهتر است. تعریف شما هم خوب است اما با محدودیت هایی مواجه است مثلا آیا می توانید بگویید این مجوعه متناهی است یا نه: { a,g,h,5,6,...,b}؟[/QUOTE]

بستگی به مقادیر a و g و h و b خواهد داشت.:46:

البته اگه تعریف دقیق‌تر قابل شمارش رو بخواید، باید یه سری به چندتا کتاب بزنم!:31:

sd70
18-11-2007, 18:31
ايناش رو خودم حل کردم (1 3 4 5 8) لطفا بقيش رو راهنمايي کنيد.

alishahbazi
20-11-2007, 21:28
سلام خدمت تمامی دوستان

دوست عزیز mofidy1 جان اگه میشه یک مثال بزنی که من متوجه بشم خیلی ممنونت میشم.
با تشکرات فراوان(علی شهبازی)

دوستان کسی به سوال ما جواب نمیده ! تو رو خدا اگه امکانش هست یک مثال برایم حل کنید.:41::41::41:

seeemesooo
21-11-2007, 16:10
لطفا فرمول محاسبه بهره بانکی وقتی بهره به اصل سرمایه اضافه میشه

the dead
21-11-2007, 20:11
قربون دستون لطف کنين همين دو تارم جواب بدين
[ برای مشاهده لینک ، لطفا با نام کاربری خود وارد شوید یا ثبت نام کنید ]
[ برای مشاهده لینک ، لطفا با نام کاربری خود وارد شوید یا ثبت نام کنید ]

pp8khat
21-11-2007, 22:43
لطفا فرمول محاسبه بهره بانکی وقتی بهره به اصل سرمایه اضافه میشه

درست منظورتون رو نفهمیدم...یعنی سوالتون رو نفهمیدم...
ولی فکر کنم جوابتون تو یکی از اینا باشه دوست عزیز:
محاسبه ی بهره ی مرکب:
[ برای مشاهده لینک ، لطفا با نام کاربری خود وارد شوید یا ثبت نام کنید ]
محاسبه بهره قسط السنین:
[ برای مشاهده لینک ، لطفا با نام کاربری خود وارد شوید یا ثبت نام کنید ]
توضیح:در صورتی که "کا اندیس n" برابر صفر باشد،فرمول های مربوط به باز پرداخت و استهلاک به دست می آید.

محاسبه ی حساب سپرده(فرمول بانکی حساب پس انداز):
[ برای مشاهده لینک ، لطفا با نام کاربری خود وارد شوید یا ثبت نام کنید ]
راهنما:
n= تعداد سال
p=نرخ بهره
q=p+1
"کا اندیس صفر"=سرمایه اولیه
"کا اندیس n"=سرمایه پس از n سال
r=برداشت های سالیانه
==============
لطفاً یکی اون سوال من رو هم جواب بده ممنون...:40:

negarine
22-11-2007, 23:11
قربون دستون لطف کنين همين دو تارم جواب بدين
[ برای مشاهده لینک ، لطفا با نام کاربری خود وارد شوید یا ثبت نام کنید ]
[ برای مشاهده لینک ، لطفا با نام کاربری خود وارد شوید یا ثبت نام کنید ]
برای سوال اولی و دومی (و سوالاتی از همین دست) بسیار راحت است که مقدار عبارت رو برابر A فرض کنیم و از A لگاریتم نپرین بگیریم.حال حد این لگاریتم رو محاسبه میکنیم و سپس مقداری که به دست میآید رو براحتی میتونیم با تابع EXP حساب کنیم.
اگه تونستین که هیچ..اگه نه خودم محاسبه کاملش رو میزارم. :20:

seeemesooo
23-11-2007, 13:17
با تشکر منظورم این بود اگر 100 تومان در بانک بگذارم و ماه بعد همان مبلع به اضافه سود ماه او ل را بگذارم بعد از n ماه چقدر دارم

abbas_kh28
29-11-2007, 14:07
اگر كسي maple بلده اين نرم افزار رو براي رسم توابع يك اموزشي بده

popeye
02-12-2007, 10:26
اساتيد لطف كنن به اين سوال جواب بدن:
كمترين فاصله ي نقطه ي (1و1) را از سهمي y=x*2+1بيابيد.
خيلي ممنون

pp8khat
02-12-2007, 14:22
اساتيد لطف كنن به اين سوال جواب بدن:
كمترين فاصله ي نقطه ي (1و1) را از سهمي y=x*2+1بيابيد.
خيلي ممنون
البته من استاد نیستم...:11:
ولی فکر کنم جواب سوال شما رو به دست آوردم(اصلاً مطمئن نیستم):
حل:
شکل صورت سوال:
[ برای مشاهده لینک ، لطفا با نام کاربری خود وارد شوید یا ثبت نام کنید ]
طبق فرمول فاصله نقطه از خط:
[ برای مشاهده لینک ، لطفا با نام کاربری خود وارد شوید یا ثبت نام کنید ]

SuB
03-12-2007, 23:37
البته من استاد نیستم...:11:
ولی فکر کنم جواب سوال شما رو به دست آوردم(اصلاً مطمئن نیستم):
حل:
شکل صورت سوال:
[ برای مشاهده لینک ، لطفا با نام کاربری خود وارد شوید یا ثبت نام کنید ]
طبق فرمول فاصله نقطه از خط:
[ برای مشاهده لینک ، لطفا با نام کاربری خود وارد شوید یا ثبت نام کنید ]

نه دوست عزیز از این راه بدست نمیاد. جواب شما 0.70710678118654752440084436210485 هست ولی من جواب کوچکتر از اونم سراغ دارم. 0.55901699437494742410229341718282 حتی از این کوچکتر هم پیدا میشه.:21:

اما راه حل::5:
اینگونه سوالها رو این طوری حل می‌کنند که می‌نویسند d برابر است با فاصله نقطه (1,1) از نقطه (x,y) روی نمودار. پس باید مقدار مینیمم این d رو بدست بیارید. برای این کار باید تابع رو یک متغیره کنید. برای همین به جای y توی عبارت مساوی d، عبارت x^2 + 1 رو جایگزین می‌کنی. بعد تابع میشه یه متغیره و با استفاده از مشتق مقدار مینیمم مطلق رو بدست میاری که همون جواب مسئله هست.

اما توی این سوال یه مشکل هست. وقتی d رو بر حسب x می‌کنی، عبارت میشه رادیکال x^4 + x^2 - 2x + 1 . روشنه که وقتی d مینیمم است که x^4 + x^2 - 2x + 1 مینیمم باشد. برای یافتن مختص اول (x) نقطه مینیمم باید ازش مشتق بگیری. مشتقش میشه 4x^3 + 2x -2 . و باید ریشه مشتق رو پیدا کنی.
همونطور که می‌بینی مشتق از درجه 3 است و می‌توان ثابت کرد یک و فقط یک ریشه دارد و آنهم بین 0 و 1 است ولی مقدار آن معلوم نیست.:18: اگه توانستید این معادله رو حل کنید:46:، پس از یافتن x اون رو در عبارت مساوی با d قرار بدید تا مقدار مینیمم فاصله بدست آید.
:11:

hitman45
05-12-2007, 19:22
با سلام
لطفا يكي سوال زير رو حل كنه و اگه ممكنه با راه حل متعدد

[ برای مشاهده لینک ، لطفا با نام کاربری خود وارد شوید یا ثبت نام کنید ]

hitman45
07-12-2007, 07:55
با سلام
لطفا يكي سوال زير رو حل كنه و اگه ممكنه با راه حل متعدد

[ برای مشاهده لینک ، لطفا با نام کاربری خود وارد شوید یا ثبت نام کنید ]

كسي نيست بتونه تو اين زمينه كمك كنه؟!

SuB
07-12-2007, 21:40
سلام
دوستان میشه با استفاده از هم‌نهشتی، دو رقم سمت راست 89 به توان 9 رو بدست بیارند و اینجا روش کار رو بنویسند؟ من هرچی زور می‌زنم، دو رقم سمت راستش میشه 9 در صورتی که باید 89 بدست بیاد!

پاکر
08-12-2007, 00:57
با سلام
لطفا يكي سوال زير رو حل كنه و اگه ممكنه با راه حل متعدد

[ برای مشاهده لینک ، لطفا با نام کاربری خود وارد شوید یا ثبت نام کنید ]

پست زیر رو بخونین شاید مفید باشه:11:

برای مشاهده محتوا ، لطفا وارد شوید یا ثبت نام کنید

Aryan.kh
08-12-2007, 15:58
سلام من تازه با این تاپیک آشنا شدم 2 تا سوال داشتم ممنون میشم اگر لطف کنید و حلش کنید
(شرمندم اما من باید جواب این 2 سوال رو حتما تا 2 شنبه ساعت 4 یا 5 صبح داشته باشم ) چون صبح 2 شنبه کلاس دارم و بد جوری احتمالش هست که در امتحان نیم ترم یکیش بیاد پیشاپیش ممنونم

[ برای مشاهده لینک ، لطفا با نام کاربری خود وارد شوید یا ثبت نام کنید ]

sickle&hammer
08-12-2007, 22:18
با عرض سلام و خسته نباشید خدمت همه :11:

من کاربر جدید هستم و نمی دونم چطور می تونم در پست هام تصویر قرار بدم ...................................لطفا کمک کنید :19:

sherlockholmz
09-12-2007, 08:39
سلام من تازه با این تاپیک آشنا شدم 2 تا سوال داشتم ممنون میشم اگر لطف کنید و حلش کنید
(شرمندم اما من باید جواب این 2 سوال رو حتما تا 2 شنبه ساعت 4 یا 5 صبح داشته باشم ) چون صبح 2 شنبه کلاس دارم و بد جوری احتمالش هست که در امتحان نیم ترم یکیش بیاد پیشاپیش ممنونم

[ برای مشاهده لینک ، لطفا با نام کاربری خود وارد شوید یا ثبت نام کنید ]

سلام،اولي را برات حل مي كنم:

برای مشاهده محتوا ، لطفا وارد شوید یا ثبت نام کنید

sherlockholmz
09-12-2007, 08:44
سلام من تازه با این تاپیک آشنا شدم 2 تا سوال داشتم ممنون میشم اگر لطف کنید و حلش کنید
(شرمندم اما من باید جواب این 2 سوال رو حتما تا 2 شنبه ساعت 4 یا 5 صبح داشته باشم ) چون صبح 2 شنبه کلاس دارم و بد جوری احتمالش هست که در امتحان نیم ترم یکیش بیاد پیشاپیش ممنونم

[ برای مشاهده لینک ، لطفا با نام کاربری خود وارد شوید یا ثبت نام کنید ]

دومي را راهنمائي مي كنم:

برای مشاهده محتوا ، لطفا وارد شوید یا ثبت نام کنید

sherlockholmz
09-12-2007, 08:47
در مورد دومي،كافيست با مخرج مشترك گيري ومساوي قرار دادن طرفين، a,b,c,d را بدست آوريم . انتگرال به چهار انتگرال تبديل ميشود :دوتا ln و دو تا tan ...بقيه اش به عهده خودت كه نمره ات حلال باشد.

Aryan.kh
09-12-2007, 09:02
خیلی ممنون sherlock holmz لطف کردی برای من واقعا جالبه که مدت هاست در p30world عضوم اما تا به حال این تاپیک رو ندیده بودم واقعا عالی هست این تاپیک امیدوارم من هم بتونم در این تاپیک حضور مستمر داشته باشم و در حد توانم به بقیه اعضا کمک کنم

Aryan.kh
09-12-2007, 09:04
با عرض سلام و خسته نباشید خدمت همه :11:

من کاربر جدید هستم و نمی دونم چطور می تونم در پست هام تصویر قرار بدم ...................................لطفا کمک کنید :19:

سلام من هم تازه واردم
ببین باید در یک برنامه که قابلیت تایپ ریاضی داره مثل word سوالت رو بنویسی و بعد با فرمت تصویری اون رو آپ لود کنی مثلا jpg

banisadr
09-12-2007, 16:58
اگه می شه روابط بین مجموع ها رو اثبات کنید

sickle&hammer
09-12-2007, 19:22
اگر ممکنه کسی این رو برای من اثبات کنه البته از راه غیر از عدد گذاری و نمایش محور اعداد (اگر از راه استقرای ریاضی باشد خیلی بهتر است)

[ برای مشاهده لینک ، لطفا با نام کاربری خود وارد شوید یا ثبت نام کنید ]

sickle&hammer
09-12-2007, 19:29
[ برای مشاهده لینک ، لطفا با نام کاربری خود وارد شوید یا ثبت نام کنید ]

banisadr
09-12-2007, 20:48
چرا کسی جواب ما رو نمیده

sherlockholmz
10-12-2007, 12:29
[ برای مشاهده لینک ، لطفا با نام کاربری خود وارد شوید یا ثبت نام کنید ]


سلام،من كه چيزي نمي بينم. حالا يا چشماي من خيلي ضعيفه يا...
اگه تونستي واضح چاپ كن تا ببينم چي مي شه.

pp8khat
10-12-2007, 17:42
اگه می شه روابط بین مجموع ها رو اثبات کنید

با نمودار ون که کارت راحت را می افته!!
دیگه چی می خوای؟؟

sickle&hammer
10-12-2007, 20:12
[ برای مشاهده لینک ، لطفا با نام کاربری خود وارد شوید یا ثبت نام کنید ]

sickle&hammer
10-12-2007, 20:13
بالاخره تونستم این عکس رو upload کنم

SuB
10-12-2007, 23:46
با نمودار ون که کارت راحت را می افته!!
دیگه چی می خوای؟؟

والا تا اونجایی که من می‌دونم، با نمودار ون هیچ چیز اثبات نمی‌شود بلکه به عنوان شهود برای درک بهتر اثبات استفاده می‌شود.:46:

SuB
10-12-2007, 23:50
سلام
دوستان میشه با استفاده از هم‌نهشتی، دو رقم سمت راست 89 به توان 9 رو بدست بیارند و اینجا روش کار رو بنویسند؟ من هرچی زور می‌زنم، دو رقم سمت راستش میشه 9 در صورتی که باید 89 بدست بیاد!

جناب پاکر این مخصوص شماست. لطفاً عنایت داشته باشید!:11:

pp8khat
11-12-2007, 14:22
والا تا اونجایی که من می‌دونم، با نمودار ون هیچ چیز اثبات نمی‌شود بلکه به عنوان شهود برای درک بهتر اثبات استفاده می‌شود.:46:

پس چه طور ما توی ریاضی 1 و ریاضیات تکمیلی - جبرانی با این روش حکم ها رو ثابت می کردیم...:2:
فکر کنم تو کتاب نوشته بود...:18:
راستی هر 3 تا معلمامون هم با این روش آ منهای ب مساوی آ اشتراک ب پریم یا ....رو ثابت می کردند...:13:
:11:

Carl
11-12-2007, 15:21
کسی راه‌حل پیدا کردن وارون y=x^3+x رو داره؟

یه تاپیک واسه‌ش دیدم کسی جواب درست و حسابی نداده بود.

sickle&hammer
11-12-2007, 16:00
کسی برای پست شماره 1095 جواب پیدا نکرد ؟؟؟؟؟؟؟؟

the dead
11-12-2007, 19:26
[ برای مشاهده لینک ، لطفا با نام کاربری خود وارد شوید یا ثبت نام کنید ]
آن جملات متناهی اند و يک چيزی شبيه اين قضيه هستند:
[ برای مشاهده لینک ، لطفا با نام کاربری خود وارد شوید یا ثبت نام کنید ]
راهنمايی خود مسأله:
[ برای مشاهده لینک ، لطفا با نام کاربری خود وارد شوید یا ثبت نام کنید ]

SuB
11-12-2007, 20:03
پس چه طور ما توی ریاضی 1 و ریاضیات تکمیلی - جبرانی با این روش حکم ها رو ثابت می کردیم...:2:
فکر کنم تو کتاب نوشته بود...:18:
راستی هر 3 تا معلمامون هم با این روش آ منهای ب مساوی آ اشتراک ب پریم یا ....رو ثابت می کردند...:13:
:11:

والا دبیر سال اول ریاضی ما که ریاضی تکمیلی اصلاً بهمون درس نداد و بجاش همون ریاضی 1 رو کار کرد. اما وقتی که ریاضی 1 کار میکرد، اصلاً با نمودار ون هیچ چیز رو برامون ثابت نکرد.
شاید دبیر شما از همه جا بی‌خبر بوده و فکر کرده نمودار ون هم اثبات میشه!

ان شاء الله یکم که بزرگتر شدید (:31:) و رفتید سال سوم، توی کتاب جبر و احتمال اصلاً می‌فهمید که اثبات برای مجموعه‌ها چگونه هستند.
معمولاً برای اثبات تساوی دو رابطه بین مجموعه‌ها، یا با استفاده از جبر مجموعه‌ها (مجموعه قضیه‌هایی که قبلاً اثبات شدند و اعمالی همچون اشتراک، اجتماع، متمم‌گیری، تفریق و ....) اط یک طرف تساوی به طرف دیگر می‌روند یا از هر دو طرف پیش می‌روند تا برای هر دو به یک عبارت برسند یا ثابت می‌کنند هر عضو از سمت راست، عضوی از سمت چپ (سمت راست زیر مجموعه سمت چپ) و هر عضو از سمت چپ، عضوی از سمت راست (سمت چپ زیر مجموعه سمت راست) است و از این دو نتیجه می‌گیرند که دو طرف برابرند. (اگه نفهمیدید چی گفتم، مشکل از شما نیست. مشکل از گفتن بنده هست!)

sherlockholmz
12-12-2007, 10:29
آن جملات متناهی اند و يک چيزی شبيه اين قضيه هستند:

راهنمايی خود مسأله:


[ برای مشاهده لینک ، لطفا با نام کاربری خود وارد شوید یا ثبت نام کنید ]

the dead
12-12-2007, 19:40
اينجا 4 تا جا رو مشکل داشتم

[ برای مشاهده لینک ، لطفا با نام کاربری خود وارد شوید یا ثبت نام کنید ]

sickle&hammer
13-12-2007, 19:50
نظرتون در مورد پست 1095 چیه ؟؟ آیا میشه اثباتش کرد ؟؟؟

Aryan.kh
14-12-2007, 17:53
سلام 2 تا سوال هست که می خواستم بهم در حلشون کمک کنید ( یا با حلش یا با راهنمایی در حلش ) ...

[ برای مشاهده لینک ، لطفا با نام کاربری خود وارد شوید یا ثبت نام کنید ]

و یک سوال دیگه داشتم برای تسلط بر ریاضیات دانشگاهی باید چه کار کنم من مهندسی می خونم نه ریاضیات محض و می خوام در درس ریاضی 1 و ریاضی 2 کاملا مسلط باشم ( مخصوصا انتگرال که واقعا سوالات سختی داره ) اگر امکانش هست راهنماییم کنید تا از چه کتابی یا منبعی استفاده کنم که سوالات خوبی داشته باشه و حدودا چه قدر در هفته وقت بزارم علاوه بر این دو کتاب به نظر شما گسسته و هندسه چه قدر در رشته IT که من برای کارشناسی ارشدمی خوام واردش بشم به درد من می خوره چون من این دو درس رو در دبیرستان و پیش دانشگاهی مشکل داشتم و می خواستم ببینم باید بر اون ها هم مسلط باشم چون می خواستم از همین ترم 1 به فکر کارشناسی ارشد و کنکورش باشم می خواستم بدونم چه طور می تونم بر این دروس مسلط باشم و کدامیک از این مباحث در کنکور کارشناسی ارشد IT لازم هست ( منظورم هندسه و گسسته هست )
پیشاپیش ممنونم

sherlockholmz
15-12-2007, 11:04
اينجا 4 تا جا رو مشکل داشتم

[ برای مشاهده لینک ، لطفا با نام کاربری خود وارد شوید یا ثبت نام کنید ]

تا "f كه مشكلي نيست.جمله اول "f ،يك عامل (x-a) دارد كه توان آن m-2 است. با مشتق گيري متوالي ودانستن مشتق ضرب چند جمله اي (مشتق جمله اول در بقيه بعلاوه مشتق جمله دوم در بقيه ...) سرانجام در مشتق m-2 به عامل (x-a) بتوان 2 مي رسيم كه در مرتبه بعد مشتق آن (x-a)با ضريب 2 خواهد بوددر بقيه جملات.همين طور جمله بعدي مشتق چون از يك مرحله بعد شروع مي شود،سرانجام به جمله(x-a) با توان 2و ضريب 3 منتهي مي گردد.و...وجمله يكي به آخر مانده چون تنها در آخرين مرحله مشتث گيري مي شود،تنها عامل(x-a) به توان m-1 و ضريب m را دارد(توجه كن كه ما بايد ثابت كنيم عامل (x-a) در تمام جملات مشترك است و كاري به ما بقي جملات نداريم.)براي درك بهتر بيا فرض كنيم جملات و توانها محدود است بصورت زير :
[ برای مشاهده لینک ، لطفا با نام کاربری خود وارد شوید یا ثبت نام کنید ]
فكر كنم با اين مثال قضيه حله، درسته؟

the dead
15-12-2007, 18:17
آقا گرفتم چی ميشه
فقط اينرو بگين که اون جملات رو مشتق نگرفتين ديگه ولی اگر ميگرفتين هم در حل مسأله تأثيری نداشته درسته

wantonboy13
15-12-2007, 22:47
با سلام ، تاپیک جالبیه... من دنبال حل یه مسئله بودم که کارم به این تاپیک کشیده شد.امیدوارم ذغال خوب باعث نشه که به این تاپیک معتاد بشم !! :27:
حالا بریم سر اصل مطلب؛
برای برنامه نویسی ما استادمون یه 10-12 تا پروژه داده که یکی از این پروژه ها بیشتر به ریاضیات و ذهن خلاق بچه ها ربط داره.
سوال اینه که:
کاربری یک عدد بین 1 تا 256 به دلخواه انتخاب می کنه ، شما باید با پرسیدن حداکثر 8 سوال به عدد مورد نظر برسید. (طوریکه جوابها فقط بله و یا خیر باشند).
ببینم دوستان متفکر و خلاقمون در این تاپیک چی کار می کننا.... رو سفیدمون کنید :21:

Aryan.kh
16-12-2007, 10:05
سلام 2 تا سوال هست که می خواستم بهم در حلشون کمک کنید ( یا با حلش یا با راهنمایی در حلش ) ...

[ برای مشاهده لینک ، لطفا با نام کاربری خود وارد شوید یا ثبت نام کنید ]

و یک سوال دیگه داشتم برای تسلط بر ریاضیات دانشگاهی باید چه کار کنم من مهندسی می خونم نه ریاضیات محض و می خوام در درس ریاضی 1 و ریاضی 2 کاملا مسلط باشم ( مخصوصا انتگرال که واقعا سوالات سختی داره ) اگر امکانش هست راهنماییم کنید تا از چه کتابی یا منبعی استفاده کنم که سوالات خوبی داشته باشه و حدودا چه قدر در هفته وقت بزارم علاوه بر این دو کتاب به نظر شما گسسته و هندسه چه قدر در رشته IT که من برای کارشناسی ارشدمی خوام واردش بشم به درد من می خوره چون من این دو درس رو در دبیرستان و پیش دانشگاهی مشکل داشتم و می خواستم ببینم باید بر اون ها هم مسلط باشم چون می خواستم از همین ترم 1 به فکر کارشناسی ارشد و کنکورش باشم می خواستم بدونم چه طور می تونم بر این دروس مسلط باشم و کدامیک از این مباحث در کنکور کارشناسی ارشد IT لازم هست ( منظورم هندسه و گسسته هست )
پیشاپیش ممنونم

ممنون میشم منو راهنمایی کنید فردا کلاس دارم ...:18:

the dead
16-12-2007, 15:41
تا "f كه مشكلي نيست.جمله اول "f ،يك عامل (x-a) دارد كه توان آن m-2 است. با مشتق گيري متوالي ودانستن مشتق ضرب چند جمله اي (مشتق جمله اول در بقيه بعلاوه مشتق جمله دوم در بقيه ...) سرانجام در مشتق m-2 به عامل (x-a) بتوان 2 مي رسيم كه در مرتبه بعد مشتق آن (x-a)با ضريب 2 خواهد بوددر بقيه جملات.همين طور جمله بعدي مشتق چون از يك مرحله بعد شروع مي شود،سرانجام به جمله(x-a) با توان 2و ضريب 3 منتهي مي گردد.و...وجمله يكي به آخر مانده چون تنها در آخرين مرحله مشتث گيري مي شود،تنها عامل(x-a) به توان m-1 و ضريب m را دارد(توجه كن كه ما بايد ثابت كنيم عامل (x-a) در تمام جملات مشترك است و كاري به ما بقي جملات نداريم.)براي درك بهتر بيا فرض كنيم جملات و توانها محدود است بصورت زير :
[ برای مشاهده لینک ، لطفا با نام کاربری خود وارد شوید یا ثبت نام کنید ]
فكر كنم با اين مثال قضيه حله، درسته؟

آقا گرفتم چی ميشه
فقط اينرو بگين که اون جملات رو مشتق نگرفتين ديگه ولی اگر ميگرفتين هم در حل مسأله تأثيری نميداشت درسته

rouhallah
17-12-2007, 00:02
با سلام
لطفاً این سوال منو جواب بدین
من میخوام جواب این انتگرال[ برای مشاهده لینک ، لطفا با نام کاربری خود وارد شوید یا ثبت نام کنید ]

رو بهم بدین البته با ذکر نوع روش انتگرال گیری و مراحل گرفتن انتگرال تا رسیدن به جواب
اگه براتون ممکنه آزمون مشتق و روش مشتق گرفتن از جواب این انتگرال هم مرحله به مرحله بهم بگین ممنون میشم آخه تو انتگرال و مشتق یکمی ضعیفم میخوام با دیدن مراحل کار خوب یاد بگیرم
با تشکر

Ehsan_1368
17-12-2007, 10:54
سلام خدمت آقای مفیدی معلم عزیز سال سوم دبیرستان خودم. از امروز منم به جمع شما بروبچ باحال میپیوندم . امیدوارم که موثر واقع بشم

popeye
17-12-2007, 23:05
sub و Pp8khatعزیز از هردوی شما بی نهایت ممنونم:40:

SuB
17-12-2007, 23:41
دوستان بهتر نیست سوالات مربوط به حساب دیفرانسیل و انتگرال رو در اتاق مربوطه توی همین بخش مطرح کنید؟

sherlockholmz
18-12-2007, 12:01
آقا گرفتم چی ميشه
فقط اينرو بگين که اون جملات رو مشتق نگرفتين ديگه ولی اگر ميگرفتين هم در حل مسأله تأثيری نداشته درسته

بله عزيزم،درسته
در اين جور مسائل مقدار و نوشتن كامل مراحل به خود فرد بستگي دارد فقط بايد بطور وضوح براي خواننده محرز شود كه راه حل درست است.
موفق باشيد.:20:

sherlockholmz
18-12-2007, 12:51
با سلام
لطفاً این سوال منو جواب بدین
من میخوام جواب این انتگرال[ برای مشاهده لینک ، لطفا با نام کاربری خود وارد شوید یا ثبت نام کنید ]

رو بهم بدین البته با ذکر نوع روش انتگرال گیری و مراحل گرفتن انتگرال تا رسیدن به جواب
اگه براتون ممکنه آزمون مشتق و روش مشتق گرفتن از جواب این انتگرال هم مرحله به مرحله بهم بگین ممنون میشم آخه تو انتگرال و مشتق یکمی ضعیفم میخوام با دیدن مراحل کار خوب یاد بگیرم
با تشکر

خدا نكنه ضعيف باشي!
كلا"،براي حل انتگرالهائي كه مخرج چند جمله ائي دارندبايدآنها را به مجموع چند كسر با مخرج چندجمله ايهاي درجه يك و دو تبديل كرد(نگران نباش ،قضيه داريم كه هميشه مي شود اين كار را كرد.)
پس از تبديل ديگر انتگرال گيري ساده است.
سوالي داشتي در خدمتم.

[ برای مشاهده لینک ، لطفا با نام کاربری خود وارد شوید یا ثبت نام کنید ]

براي حل انتگرال دوم يك تغيير متغير لازم است:

[ برای مشاهده لینک ، لطفا با نام کاربری خود وارد شوید یا ثبت نام کنید ]


براي تست صحيح بودن انتگرال ،كا فيست از طرف دوم مشتق بگيري تا طرف اول بدست آيد.(بگيرتايادبگيري!)
موفق باشي:31:

sherlockholmz
18-12-2007, 12:55
يك سوال ،انتگرال زير را محاسبه كنيد و يا ثابت كنيد كه انتگرال ندارد:
[ برای مشاهده لینک ، لطفا با نام کاربری خود وارد شوید یا ثبت نام کنید ]

سعید رسولی
18-12-2007, 14:03
:biggrin: برای تنوع این سوال هم بد نیست:
n نفر در یک اتاق قرار دارند. به چند حالت این n نفر می توانند با هم دوست شوند.
( :) از دوستان خواهش میکنم اگر کسی نمونه این سوال را قبلا مشاهده کرده به من اطلاع دهد)
به نظر من:!n *n

rouhallah
18-12-2007, 23:41
سوالي داشتي در خدمتم.


خیلی ممنون

rouhallah
19-12-2007, 12:34
خدا نكنه ضعيف باشي!


راستی من تو این روش این ضعف رو دارم که چطوری تعیین میکنید A و B , هر چیز دیگه ای که بستگی به تعداد کسرها داره رو مقدار میدین
مثلاً شما چطوری به B مقدار صفر رو دادین این صفر از کجا میاد یا یک دومی که به A نسبت دادید

sherlockholmz
19-12-2007, 17:00
راستی من تو این روش این ضعف رو دارم که چطوری تعیین میکنید A و B , هر چیز دیگه ای که بستگی به تعداد کسرها داره رو مقدار میدین
مثلاً شما چطوری به B مقدار صفر رو دادین این صفر از کجا میاد یا یک دومی که به A نسبت دادید

خوب ما كسر اول را بسط داديم. پس از مخرج مشترك گرفتن از كسر دوم، در صورت طرف دوم اين چند جمله اي بدست آمد:

[ برای مشاهده لینک ، لطفا با نام کاربری خود وارد شوید یا ثبت نام کنید ]

كه بايد با صورت كسر اصلي مساوي باشد.اما صورت كسر اصلي 1 است. پس كليه ضرائب x بايد برابر با صفر بوده و همچنين مقدار ثابت 2A بايد برابر با 1 باشد.با حل اين دستگاه چهار ضريب A,B,C و D بدست مي آيد.مثلا" 2A=1 پس A=1/2 و بهمين ترتيب...

sherlockholmz
19-12-2007, 17:02
يك سوال ،انتگرال زير را محاسبه كنيد و يا ثابت كنيد كه انتگرال ندارد:
[ برای مشاهده لینک ، لطفا با نام کاربری خود وارد شوید یا ثبت نام کنید ]

آيا كسي ميتواند كه اين انتگرال را حل كند؟

Aryan.kh
19-12-2007, 18:31
سلام 2 تا سوال هست که می خواستم بهم در حلشون کمک کنید ( یا با حلش یا با راهنمایی در حلش ) ...
[ برای مشاهده لینک ، لطفا با نام کاربری خود وارد شوید یا ثبت نام کنید ]

encarta
19-12-2007, 23:15
با سلام کسی هست این سوال را حل کنه ؟

مثلث Abc را که دارای زاوایای حاده است در نضر میگیریم نیمساز داخلی زاویه ی ش را رسم می کنیم تا ضلع Bc را د نقطه ی L و دایره ی محیطی مثلث را در نقطه ی N قطع کند . از L عمود هایی بر اضلاع Ab و Ac رسم میکنیم و پای دو عمود راk و M می نامیم . ثابت کنید مساحت چهارضلعی Aknm با مساحت مثلث Abc برابر است.

encarta
19-12-2007, 23:18
با سلام کسی هست این سوال را حل کنه ؟

مثلث Abc را که دارای زاوایای حاده است در نضر میگیریم نیمساز داخلی زاویه ی ش را رسم می کنیم تا ضلع Bc را د نقطه ی L و دایره ی محیطی مثلث را در نقطه ی N قطع کند . از L عمود هایی بر اضلاع Ab و Ac رسم میکنیم و پای دو عمود راk و M می نامیم . ثابت کنید مساحت چهارضلعی Aknm با مساحت مثلث Abc برابر است.

Aryan.kh
22-12-2007, 00:48
کسی جوابی برای پست 1125 پیدا نکرد

the dead
24-12-2007, 22:09
آيا كسي ميتواند كه اين انتگرال را حل كند؟


[ برای مشاهده لینک ، لطفا با نام کاربری خود وارد شوید یا ثبت نام کنید ]

Polylogarithm
The polylogarithm (also known as de Jonquière's function) is a special function Lis(z) that is defined by the sum
[ برای مشاهده لینک ، لطفا با نام کاربری خود وارد شوید یا ثبت نام کنید ]

It is in general not an elementary function, unlike the related logarithm function. The above definition is valid for all complex numbers s and z where |z|< 1. The polylogarithm is defined over a larger range of z than the above definition allows by the process of analytic continuation.
The special case s = 1 involves the ordinary natural logarithm (Li1(z)=-ln(1-z)) while the special cases s = 2 and s = 3 are called the dilogarithm (also referred to as Spence's function) and trilogarithm respectively. The name of the function comes from the fact that it may alternatively be defined as the repeated integral of itself, namely that
دیدی که میشه

sherlockholmz
25-12-2007, 10:45
[ برای مشاهده لینک ، لطفا با نام کاربری خود وارد شوید یا ثبت نام کنید ]

Polylogarithm
The polylogarithm (also known as de Jonquière's function) is a special function Lis(z) that is defined by the sum
[ برای مشاهده لینک ، لطفا با نام کاربری خود وارد شوید یا ثبت نام کنید ]

It is in general not an elementary function, unlike the related logarithm function. The above definition is valid for all complex numbers s and z where |z|< 1. The polylogarithm is defined over a larger range of z than the above definition allows by the process of analytic continuation.
The special case s = 1 involves the ordinary natural logarithm (Li1(z)=-ln(1-z)) while the special cases s = 2 and s = 3 are called the dilogarithm (also referred to as Spence's function) and trilogarithm respectively. The name of the function comes from the fact that it may alternatively be defined as the repeated integral of itself, namely that
دیدی که میشه
باسلام و تشكر،
ممنون از پستت اما...(اين اماش كاررا خراب مي كند)
نكته مهم اينست كه آيا اين انتگرال در محدوده اعداد حقيقي جواب دارد يا خير؟
با اين حساب و بدست آوردن اين جواب خفن، پس جواب حقيقي ندارد؟ok؟
حال يك سوال ديگر:

[ برای مشاهده لینک ، لطفا با نام کاربری خود وارد شوید یا ثبت نام کنید ]



آيااين انتگرال قابل حل است؟
ممنون

the dead
26-12-2007, 10:08
سلام 2 تا سوال هست که می خواستم بهم در حلشون کمک کنید ( یا با حلش یا با راهنمایی در حلش ) ...
يک چيز مشخصی است که انتگرال اولی در محدوده اعداد حقيقی قابل حل نيست و بايد در COMPLEX حل شود
حل اين نوع انتگرالها در کتاب رياضيات مهندسی توضيح داده شده[ برای مشاهده لینک ، لطفا با نام کاربری خود وارد شوید یا ثبت نام کنید ]

2n.m
28-12-2007, 23:51
سلام والا یه سوال المپیادی داشتم که نمیدونم که دارم تو جای مناسبی مطرحش ممی کنم یا نه اگه نه ببخشید چون من به این قسمت اشنایی ندارم
سوال=چند نقطه داریم که روی یک خط راست واقع نیستند وروی هریک از انان عددی نوشته شده است اگر خطی راست ازنقاط عبور کند تمامی انان حاصلی برابر صفر دارند ثابت کنید که همه ی نقاط مساوی صفر هستند؟

zahedy2006
29-12-2007, 00:01
سلام والا یه سوال المپیادی داشتم که نمیدونم که دارم تو جای مناسبی مطرحش ممی کنم یا نه اگه نه ببخشید چون من به این قسمت اشنایی ندارم
سوال=چند نقطه داریم که روی یک خط راست واقع نیستند وروی هریک از انان عددی نوشته شده است اگر خطی راست ازنقاط عبور کند تمامی انان حاصلی برابر صفر دارند ثابت کنید که همه ی نقاط مساوی صفر هستند؟


باور کن اصلا منظور سوال را نفهمیدم برای همین ممکنه جواب اشتباه باشه

اگه درست فهمیده باشم تعدادی نقطه داریم که روی همه انها تعدادی عدد نوشته شده است و هر خط راستی که بکشیم مجموع دو عدد صفر می شود.

بین هر دو نقطه ، حتما یک خط راست می توان کشید. اگر عدد روی نقطه انتخابی اول x باشد باید روی نقطه انتخابی دوم x- باشد تا حاصل صفر شود. چون ممکن است دو نقطه ای که انتخاب می شوند هر دو x باشند پس حاصل 2x می شود که حاصل باید صفر باشد یعنی x=0 پس x- هم صفر است

اصلا فکر نمی کنم که منظور را درست گرفته باشم چون این سوال شبیه المپیادی ها نیست..

encarta
29-12-2007, 12:41
سلام 2 تا سوال هست که می خواستم بهم در حلشون کمک کنید ( یا با حلش یا با راهنمایی در حلش ) ...
[ برای مشاهده لینک ، لطفا با نام کاربری خود وارد شوید یا ثبت نام کنید ]



فکر کنم جوابش این باشه


[ برای مشاهده لینک ، لطفا با نام کاربری خود وارد شوید یا ثبت نام کنید ]

Bad_Sector
30-12-2007, 18:07
با سلام خدمت دوستان پی سی ورلدی .
من یه سوال دارم اگه میشه جواب بدید چون واقعا لازم دارم .

1. مجموعه جواب نامعادله زیر را بدست آورید .
[ برای مشاهده لینک ، لطفا با نام کاربری خود وارد شوید یا ثبت نام کنید ]

Aryan.kh
30-12-2007, 19:24
فکر کنم جوابش این باشه



[ برای مشاهده لینک ، لطفا با نام کاربری خود وارد شوید یا ثبت نام کنید ]

اگر ممکنه در جای دیگری آپ لود کنید چون باز نمیشه :19:
ممنونم از لطفتون :11:

sherlockholmz
31-12-2007, 08:52
[ برای مشاهده لینک ، لطفا با نام کاربری خود وارد شوید یا ثبت نام کنید ]
با سلام خدمت دوستان پی سی ورلدی .
من یه سوال دارم اگه میشه جواب بدید چون واقعا لازم دارم .

1. مجموعه جواب نامعادله زیر را بدست آورید .
[ برای مشاهده لینک ، لطفا با نام کاربری خود وارد شوید یا ثبت نام کنید ]

aminkarami
02-01-2008, 13:51
سلام سواله من نسبتا آسونه می خواستم بدونم جوابه این چی میشه در موردش با دبیرمون به مشکل خوردم.
[ برای مشاهده لینک ، لطفا با نام کاربری خود وارد شوید یا ثبت نام کنید ]
خواهشا کمکم کنید چون به مشکل برخوردم
مرسی امین

zahedy2006
02-01-2008, 14:25
فکر کنم با دوتا نکته به راحتی حل می شود

1) جزء صحیح همواره عدد است و در مشتق گیری عدد به حساب می آید (یعنی مشتق آن صفر است)

2) وقتی که درون جزء صحیح به سمت بی نهایت میل می کند می توان از جزء صحیح صرف نظر کرد.

فکر کنم جواب از راست و چپ بشود 2

smahdis
02-01-2008, 20:16
دوستان خواهش ميكنم كمكم كنيد اين مسئله رو حل كنيد مربوط به هندسه 1 سال دوم دبيرستان هست:

سئوال: ارتفاع مثلثي نصف قاعده آن است. اگر مساحت مثلث 36 متر مربع باشد طول قاعده آن را پيدا كنيد.

من فردا امتحان هندسه پايان ترم دارم. خواهش ميكنم اگر ميتونيد اين رو حلش كنيد تا فردا.

pp8khat
03-01-2008, 10:17
سئوال: ارتفاع مثلثي نصف قاعده آن است. اگر مساحت مثلث 36 متر مربع باشد طول قاعده آن را پيدا كنيد.
اولاً:سوال
ثانیاً:
جواب نمی دم!!!:41::41::41:
ویرایش(نظرم عوض شد):
یعنی شما تو کجای این مساله موندید؟؟
هر چی به فارسی نوشته به زبان ریاضی بنویسید...
h=d/2
d=2h
dh/2=36
2h^2=72
h^2=36
h=6
d=12
نکته:در هندسه مسطحه اندازه پاره خط هرگز منفی نمی شود...

mass0ood
03-01-2008, 23:32
سلام به دوستان گل و علم دوست خودم؛ البته اگه من رو به عنوان یک دوست قبول کنید...
من از دوستانی که تند تند میان مسئله می ذارن تو فروم بعد می گن حلش کنید یه خواهش دارم:
یک کم بیشتر رو مسئله هاتون فکر کنین و سعی کنید خودتون به جواب برسید؛ نه این که یه نفر دیگه بیاد برای شما حل کنه...این طوری قدرت محاسباتتون ضعیف می شه و بعد دچار مشکل می شید. مخصوصا بچه های سال اول و دوم و سوم دبیرستان که خودمم سومم :d
البته اگه واقعا روش فکر کردید و به جواب نرسیدید مطرحش کنید.
ببخشید که من اول ورودم این چیزا رو گفتم. اما خوب به خاطر خودتونه.

sanih
16-01-2008, 00:23
دوستان جواب این چی می شه
[...0.999]
1)0 2)1 3)هیچ کدام 4) ...نمدونم چون اصلا مهم نیست!

sanih
16-01-2008, 00:24
گزینه ی دوم یعنی 1 جواب است.

براکت ، براکت کفه.

sanih
16-01-2008, 00:25
ببخشد خیلی سطح پایین بود ولی خیلی ها رو گول زده.

نکنه قبلا مطرح شده؟

SuB
16-01-2008, 11:10
دوستان جواب این چی می شه
[...0.999]
1)0 2)1 3)هیچ کدام 4) ...نمدونم چون اصلا مهم نیست!

گزینه 2:31:

sanih
17-01-2008, 01:52
subجان خنده دار بود؟:47:

number_23
17-01-2008, 13:33
دوستان جواب این چی می شه
[...0.999]
1)0 2)1 3)هیچ کدام 4) ...نمدونم چون اصلا مهم نیست!

جواب درست گزینه ی 1 هست یعنی 0 منظور از جزء صحیح یعنی کوچکترین عدد صحیح نزدیک به این عدد و کوچکترین عدد صحیح 0 است. اگه می گفت [0.999-] انوقت جواب 1- بود.:20:

sanih
17-01-2008, 15:50
...................

sanih
17-01-2008, 15:57
ای بابا تو هم که گول خوردی .جواب همون گزینه دومه چون:

این علامت "..." در ریاضیات به معنیه حد گرفتنه(پاورقی کتاب دیفرانسیل یا حسابان) پس اول باید ببینیم حدود این عدد چنده معلوه که 1 میشه و همه هم اینو میدونیم! که جواب حد مطلقه پس اگه براکت نبود فقط گیج نمی شدیم. پس جواب همو کزینه ی دومه!

امید وارم مفید بوده باشه. با حق.

SuB
17-01-2008, 18:35
subجان خنده دار بود؟:47:

نه. بعد از حل یه مسئله به ظاهر ساده ریاضی، خنده می‌چسبه!!!!:27::31:

sanih
18-01-2008, 01:08
نه. بعد از حل یه مسئله به ظاهر ساده ریاضی، خنده می‌چسبه!!!!:27::31:

ok ببخشید من فکر کردن خندت منظور دار بود!

به نظر من یه مسئله بذارید تا همه روش فکر کنیم نه اینکه قبلادیده باشیم به نظر من مسائل المپیاد مناسبه تو این تاپیک فضای مناسبی برای این کار است.

sanih
18-01-2008, 02:41
اگر شش نقطه بو طور تصادفي روي محيط يک دايره انتخاب کنيم.احتمال اينکه مثلثي که به وسيله سه نقطه اول ساخته مي شود با مثلثي که توسط سه نقطه دوم ساخته ميشود نطقه بخورد نداشته باشند چقدر است؟
سئوال جالبيه يکم تمرکز ميخواد.
1983

sanih
18-01-2008, 02:43
اگر شش نقطه بو طور تصادفي روي محيط يک دايره انتخاب کنيم.احتمال اينکه مثلثي که به وسيله سه نقطه اول ساخته مي شود با مثلثي که توسط سه نقطه دوم ساخته ميشود نطقه بخورد نداشته باشند چقدر است؟
سئوال جالبيه يکم تمرکز ميخواد.
1983

mohammad96
18-01-2008, 02:57
ای بابا تو هم که گول خوردی .جواب همون گزینه دومه چون:

این علامت "..." در ریاضیات به معنیه حد گرفتنه(پاورقی کتاب دیفرانسیل یا حسابان) پس اول باید ببینیم حدود این عدد چنده معلوه که 1 میشه و همه هم اینو میدونیم! که جواب حد مطلقه پس اگه براکت نبود فقط گیج نمی شدیم. پس جواب همو کزینه ی دومه!

امید وارم مفید بوده باشه. با حق.

سلام!
من فكر كنم چون حد،حد چپه ، جواب همون گزينه يك ميشه!

sanih
18-01-2008, 03:39
سلام!
من فكر كنم چون حد،حد چپه ، جواب همون گزينه يك ميشه!


نه عزیزم اینجا اصلا بحث چپ و راست نیست چون ما اصلا نموداری نداریم که بخوایم از چپ یا از راست به نقطه ای نزدیک بشیم. من سئوالم از شا اینه که حدودا اون عدد کزایی چنده؟ جواب شما معلومه یکه. یا بهتر توضیح بدم این عدد دائما داره به یک نزدیک میشه حتی همین الان اگرر بجای سه نقطه مینوستم nتا عدد اون وقت گزینه ی صحیح گزینه یک بود ولی این عدد حتی از اپسیلون هم به یک نزدیک تره واسه همین باید حد گریم .

متوجه شدی دوست عزیز یا بیشتر توضیح بدم.

sanih
18-01-2008, 18:30
آيا 1101عدد صحيح متفاوت وجود دارد که جمع مربعاتشان مکعب و جمع مکعباتشان مربع؟

sanih
18-01-2008, 18:31
همه ريشه هاي حقيقي معادله درجه 4 زير را دقيقا تا 4 رقم اعشار بيابيد:
(x^4)-(2n+1)(x^2)-x-(n^2)+n-1=0
n=10^10

sanih
18-01-2008, 18:32
فرض کنيد abcd يک چار ضلعي محاط باشد.نشان دهيد
|ab-cd|+|bc-ad|>=2|bd-ac|

sanih
18-01-2008, 18:32
سئوال هاي جالبي براي بحث گذاشتم و سعي کردم طوري باشه که هر کسي با توجه به تخصص خودش بتونه حد اقل مسئله رو يکم تکون بده.
اگه کسي حل نکرد خودم جواب هارو ميذارم
يا حق.

zahedy2006
18-01-2008, 20:20
اگر شش نقطه بو طور تصادفي روي محيط يک دايره انتخاب کنيم.احتمال اينکه مثلثي که به وسيله سه نقطه اول ساخته مي شود با مثلثي که توسط سه نقطه دوم ساخته ميشود نطقه بخورد نداشته باشند چقدر است؟
سئوال جالبيه يکم تمرکز ميخواد.
1983


با توضیح می نویسم چون شکل نمی تونم بکشم
شما ه نقطه تصادفی انتخاب کرده و مثلثی رسم می کنید. حالا نقطه چهارم بین هر کدام از این سه نقطه می تواند باشد. تنها حالتی که مثلث دوم ، مثلث اول را قطع نمی کند این است که سه نقطه دوم ، در بین دو نقطه متوالی از مثلث اول باشند که میشه 1/3 * 1/3 یعنی 1/9 (یک سوم در یک سوم = یک نهم)


آيا 1101عدد صحيح متفاوت وجود دارد که جمع مربعاتشان مکعب و جمع مکعباتشان مربع؟


خودتون بخونید اگه متوجه شدید خوبه

sanih
19-01-2008, 00:26
زاهدی جان مرسی.

جوابت کاملا غلطه جواب صحیح 0.3 میشه
سئوال بعدی هم کاملا درسته با دقت بخون.

mohammad96
19-01-2008, 00:58
نه عزیزم اینجا اصلا بحث چپ و راست نیست چون ما اصلا نموداری نداریم که بخوایم از چپ یا از راست به نقطه ای نزدیک بشیم. من سئوالم از شا اینه که حدودا اون عدد کزایی چنده؟ جواب شما معلومه یکه. یا بهتر توضیح بدم این عدد دائما داره به یک نزدیک میشه حتی همین الان اگرر بجای سه نقطه مینوستم nتا عدد اون وقت گزینه ی صحیح گزینه یک بود ولی این عدد حتی از اپسیلون هم به یک نزدیک تره واسه همین باید حد گریم .

متوجه شدی دوست عزیز یا بیشتر توضیح بدم.

سلام!
براي وجود حد يك تابع در يك نقطه (تا جايي كه معلوماتم اجازه ميده) حد چپ و راست تابع در اون نقطه بايد با هم برابر باشند. تابع [x] ، در نقاط صحيح حد ندارند. براي محاسبه حدود چپ و راست لزومي به وجود نمودار هم نيست و ضمنا فرقي نمي كنه تا چه اندازه مثلا از سمت چپ به يك عدد نزديك ميشويم، مساله اينه كه مقدار حد هميشه از اون عدد كمتره.

sanih
19-01-2008, 02:48
محمد جان مرسی مه پیگیری میکنی.

ببین عزیزم حد گرفتن یه مفهوم کلی که میشه اونو به توایع هم نسبت بدیم یعنی ما هر وقت از حد یا حدود صحبت می کنیم نباید حتما یه تابع داشته باشیم که توش xباشه. مثلا شما میخواید از کسی برای خرید چیزی پول بگرید اون طرف به شما میگه چقدر پول می خوای شما هم میگی حدودا 200000تومن حالا شما حد چپ گرفتی یا راست؟ هیچ کدوم شما حدودا گفتی 2000000 تومن منظور از حد در اینجا اینه واسه همین این سئوال تو کنکور آزمایشی مطرح شده.

باز هم بابت پی گیریت ممنونم .
یاحق

mohammad96
19-01-2008, 02:58
همه ريشه هاي حقيقي معادله درجه 4 زير را دقيقا تا 4 رقم اعشار بيابيد:
(x^4)-(2n+1)(x^2)-x-(n^2)+n-1=0
n=10^10

با سلام!
جوابها :
155377.3974
-155377.3974

منظورتون از طرح اين سوال چي بود؟

mohammad96
19-01-2008, 03:05
محمد جان مرسی مه پیگیری میکنی.

ببین عزیزم حد گرفتن یه مفهوم کلی که میشه اونو به توایع هم نسبت بدیم یعنی ما هر وقت از حد یا حدود صحبت می کنیم نباید حتما یه تابع داشته باشیم که توش xباشه. مثلا شما میخواید از کسی برای خرید چیزی پول بگرید اون طرف به شما میگه چقدر پول می خوای شما هم میگی حدودا 200000تومن حالا شما حد چپ گرفتی یا راست؟ هیچ کدوم شما حدودا گفتی 2000000 تومن منظور از حد در اینجا اینه واسه همین این سئوال تو کنکور آزمایشی مطرح شده.

باز هم بابت پی گیریت ممنونم .
یاحق

سلام!
ممنون از جوابتون. من متوجه حرف شما شدم، ولي سوالي كه مطرح شده بود يه سوال رياضي بود.من متوجه اين نمي شم كه چرا با اين سوال برخورد رياضي نمي كنيد.

sanih
19-01-2008, 19:02
محمد جان سئوال معادله خیلی سخت تر از اونیه که تو فکر میکنی جواب اشتباه هستند.انشا الله جواب ها درست رو آخر هفته میذارم.

یاحق

Blossom
21-01-2008, 22:45
آقا یه سوال!
چطور می‌شه وارون یک ماتریس n در n رو حساب کرد؟ توی ویکی پدیا یه چیزایی دیدم ولی نفهمیدمشون!
البته من یه رابطه ی درست درمون می‌خوام که با کمک اون رابطه یه برنامه بنویسم!
ـــــــــــــــــــــــــ ـــــــــــــ
*- طبق قوانین صفحه‌ی اول این تاپیک فکر کنم سوالم رو جای درستی مطرح کردم.
*- می‌شه گفت که ریاضی پدر نرم افزاره!
*- ماتریس معادل آرایه در برنامه نویسیه! ولی نسخه‌ی ضعیف آرایست! (شاید هم من ماتریس رو خوب نمی‌شناسم!)

sanih
22-01-2008, 22:57
blossom جان می تونی تو متلب بری mفایلشو کپی کنی یا اینکه از الگوریتمش ایده بگیری.

شوخی:البته اگر لقمه آماده دوست داری که....

یاحق

Blossom
23-01-2008, 18:28
blossom جان می تونی تو متلب بری mفایلشو کپی کنی یا اینکه از الگوریتمش ایده بگیری.

شوخی:البته اگر لقمه آماده دوست داری که....

یاحق

آقا من اصلا متلب کار نکردم . صد البته ندارمش.
بعد این که من فقط پرسیدم که چطور یه دترمینان و معکوس یه ماتریس n در n رو گیر بیارم!

zahedy2006
23-01-2008, 20:55
آقا من اصلا متلب کار نکردم . صد البته ندارمش.
بعد این که من فقط پرسیدم که چطور یه دترمینان و معکوس یه ماتریس n در n رو گیر بیارم!


منظورتون با دست است و به صورت محاسبه دستی
برای اینکار باید ابتدا دترمینان گیری را بلد باشید
بعد باید وارون ماتریس ترانهاده را بلد باشید
بعد می توانید دترمینان گیری کرد

سعی می کنم با عکس بگذارم گرچه در اینترنت زیاده. دانشنامه هم هست

Blossom
24-01-2008, 00:21
منظورتون با دست است و به صورت محاسبه دستی
برای اینکار باید ابتدا دترمینان گیری را بلد باشید
بعد باید وارون ماتریس ترانهاده را بلد باشید
بعد می توانید دترمینان گیری کرد

سعی می کنم با عکس بگذارم گرچه در اینترنت زیاده. دانشنامه هم هست

شما با دست بفرمایید! من با فکر الگوریتم ازش می‌سازم!
من دترمینان، ترانهاده و وارون ماتریس های 3*3 رو بیشتر بلد نیستم! اگر هم منبعی دارین که خوب توضیح داده (بهتره به زبون فارسی باشه) که فقط منبع رو بگین و زحمت نیفتین.
البته تو ویکی پدیا هم یه چیز دیدم ولی حالیم نشد!

sanih
25-01-2008, 01:28
دوستان این هم جواب سئولات:

Blossom
25-01-2008, 01:29
دوستان این هم جواب سئولات:

کدوم جواب؟

sanih
25-01-2008, 01:36
جواب سئوال احتمال:
[ برای مشاهده لینک ، لطفا با نام کاربری خود وارد شوید یا ثبت نام کنید ]

sanih
25-01-2008, 01:41
جواب سئوال معادله من که خیلی دوسش دارم!:
[ برای مشاهده لینک ، لطفا با نام کاربری خود وارد شوید یا ثبت نام کنید ]

sanih
25-01-2008, 01:44
جواب سئوال جمع اعداد....:
[ برای مشاهده لینک ، لطفا با نام کاربری خود وارد شوید یا ثبت نام کنید ]

sanih
25-01-2008, 01:48
دوستان ببخشید که اینطور نوشتم باور کنید سر کلاس زبان معلم 20 دقیقه دیر کرد من تو اون فرصت نوشتم الان هم که درسام تموم شد اومدم آپ کردن. توروخدا برام دعا کنید فردا تو المپیاد مرحله اول ریاضی قبول بشم. امروز فیزیک بود با معلمم که سئولات رو حل کردم من از تستی ها حدود 81.4%جواب داده بودم تشریحی ها هم 5 تاش درست بود یعنی 100% قبولم ولی ریاضی یه چیز دیگس!

mohammad96
26-01-2008, 00:51
با سلام!
در مورد سوال معادله درجه چهار sanih عزیز: از ایشون خواهش می کنم که یکبار دیگه سر فرصت مساله رو حل کنن چون اشتباهاتی در طی مسیر حل مرتکب شدن. مثلا در معادلات آخر بر حسب k ، بسط معادله به یک معادله درجه 4 منجر میشه! که مساله یه مقدار از اولش پیچیده تر میشه. مقدار تابع اولیه به ازای جوابهای بدست اومده تقریبا میشه منفی دو ضربدر ده به توان بیست !!

sanih
26-01-2008, 23:50
سلام دوستان میدونم تا اینکه قانون علاملت descartes رو توضح ندم خیلی مبهمه امشب وقت نمیشه ولی قول میدم بنویسم بذارم اونم سر کلاس شیمی باید بنویسم چون خودم فعلا ذهنم درگیر یه تحقیقه در مورد ترکیباته که اگه تازه یه ماه شروع کردم مو به مو آنالیزش کردم تا انشا الله یه چیز قابل تامل برای دانش آموزان و اساتید باشه!

فعلا یا حق

sanih
30-01-2008, 00:36
سلام دوباره پر بار با یه سئوال توپ اومدم البته به شرطی که خودتون حلش کنید ولی به هر حال من جواب کامل با توضیحات رو مذارم.

5 پروفسور در کنفرانس به بررسی یک موضوع می پردازند،در این کنفرانس دقیقا هر پروفسور2بار به خواب میرود برای هر جفت پروفسور لحظه ای وجود دارد که هر دو در خوابند.
نشان دهید لحظه ای وجود دارد که سه پروفسور همزمان در خواب هستند!


راهنمایی:از اصل لانه کبوتری حل میشه.
منتظر جواب های درست شما هستم.

یا حق.

parsa90p
01-02-2008, 18:30
سلام. مى خواستم ثابت كنيد كه e به توان ∏ بزرگ تر است از ∏ به توان e

parsa90p
01-02-2008, 18:32
سلام. مى خواستم ثابت كنيد كه e به توان ∏ بزرگ تر است از ∏ به توان e

sanih
04-02-2008, 00:42
فردا شب جوایشو میذارم.
راهنمایی:از تعریف ریاضی به جای نماد استفاده من.

mhm5000
06-02-2008, 20:54
یک شمارنده ۳ رقمی داریم که با هر بار زدن دکمه آن عدد آن یک واحد افزایش می یابد.

عددروی شمارنده ابتدا ۲۳۳است.دکمه آن را ۶۹۳بار می زنیم تا عدد شمارنده برابر ۹۲۶شود.این ۳ رقم شمارنده در مجموع چندبار تغییر کرده اند؟

۱)۶۹۳

۲)۷۰۳

۳)۷۶۹

۴)۷۷۲

۵)۸۲۷

جواب های خود را به صورت تشریحی وارد کنید...

با تشکر

hlpmostafa
07-02-2008, 13:53
سلام
یه سوال هندسه
یک مثلث قائم الزاویه داریم. که یک بار یک مربع به مساحت441 به صورتیکه دو ضلعش روی دو ساق مثلث و راس آن روی وتر مثلث باشد و بار دیگر یک مربع به مساحت 440 به صورتیکه یک ضلعش روی وتر مثلث باشد و دو راس آن روی دو ساق مثلث باشد حالا مجموع دو ساق کدام است؟
درمربعی به ضلع a چهار ربع دایره به شعاع a محاط کرده ایم مساحت قسمت مشترک در این چهار ربع دایره را بر حسب a بیابید
چگونه می توان یک زاویا ی نا مشخص را به سه قسمت مساوی با کمک پرگار و خطکش غیر مدرج تقسیم کرد؟

101170
13-02-2008, 20:08
[ برای مشاهده لینک ، لطفا با نام کاربری خود وارد شوید یا ثبت نام کنید ]

sherlockholmz
18-02-2008, 09:28
سلام،
شكل زير را در نظر مي گيريم:

[ برای مشاهده لینک ، لطفا با نام کاربری خود وارد شوید یا ثبت نام کنید ]

قطر مستطيل را بدو صورت محاسبه مي كنيم:

[ برای مشاهده لینک ، لطفا با نام کاربری خود وارد شوید یا ثبت نام کنید ]


يعني چه؟
يعني اينكه اين خطي كه به دليل خطاي ديد بعنوان قطر مستطيل در نظر گرفتيم يك خط راست نيست و شكسته است وهمين موضوع تفاوت مساحتها را تعيين مي كند.
نمي خواهد زياد تشويق كني!چون اين مسئله را قبلا" حل كرده بودم.

101170
18-02-2008, 17:32
آفرين خيلي خوب بود
راه حل منطقي و در عين حال ساده ايه

101170
20-02-2008, 18:13
مجلس هشتم 285 كرسي دارد . اگر سه حزب در انتخابات شركت كنند به چند حالت ممكن است هيچ حزبي اكثريت مطلق را به دست نياورد؟(يعني تعداد نمايندگان منتخب از آن حزب بيش از نصف نباشند.)


نياز فوري !!!!

101170
20-02-2008, 18:14
به چند طريق مي توان پنج مهره سياه و 5 مهره سفيد را در يك رديف قرار داد كه دقيقا سه بلوك سياه داشته باشد؟
يك بلوك سياه چند مهره سياه كنار هم است كه در دو طرف آن ها مهره سياه ديگري نباشد.

مثلا آرايش روبرو دو بلوك سفيد و سه بلوك سياه دارد. ●○○○○●●○●●

نياز فوري!!!

101170
20-02-2008, 18:17
الاغي مي خواهد از نقطه A به نقطه B برود ولي دقيقا در وسط راه درياچه اي دايره اي شكل به قطر دو كيلومتر قرار دارد . اگر فاصلهa وb چهار كيلو متر باشد كوتاه ترين مسير چند كيلو متر است؟

A. ○ .B

نياز فوري!!!!!

trainercodes
21-02-2008, 19:35
اقای 101170 شما هم در مرحله ی اول المپیاد شرکت کردین؟
فکر کنم جواب اون سوال گزینه ی 4 بود

حالا به این سوال من جواب بدین:«تعداد جواب های معادله ی زیر را در مجموعه ی اعداد طبیعی بیابید»

...........................6N-1.............6N...........6N+ 1 .................................................. ........................
.................................................. X ...............+.......Y ...........=....Z................................. .................................................. .................

(Xبه توان 6N-1به اضافه ی Yبه توان 6Nمساوی است باZبه توان 6N+1)
توجه کنید که مجهول های این معادله X,Y,Zهستند و ما باید برای مقادیر مختلف N بحث کنیم

Moh3en_DDD
25-02-2008, 00:11
فکر کنم این 2 تا سوال هم به این تاپیک مربوط باشه :


برای مشاهده محتوا ، لطفا وارد شوید یا ثبت نام کنید

shabgard10
25-02-2008, 21:40
در مورد مسئله ی الاغ باید بگم جوابش همون گزینه ی 4 میشه
البته میتونید پاشخنامه رو هم از سایت دانش پزوهان بگیرید

101170
26-02-2008, 14:14
پاسخنامه رو از سايت گرفتم
جواب تشريحيش رو مي خوام

khatarat
29-02-2008, 08:37
تو محور مختصات سه بعدی سه نقطه روی هر کدوم از محور های x y z داریم که مختصات اونها رو داریم یه نقطه هم تو فضا وجود داره که فاصله اون رو تا سه نقطه ای که روی محور ها هستند داریم و اینجا مختصات نقطه ای که تو فضاست مجهوله چطوری می تونیم مختصات x y z اون رو پیدا کنیم.
هر کی بلده زود جواب ما رو بده چون خیلی لازمش دارم.
اگر می تونید یه فرمول کلی برای این مسئله پیدا کنید که بشه با اون مختصات نقطه مجهول رو بدست آورد لطفا جواب اون رو در صورت امکان به این آدرس ارسال کنید:
khatarat31.5@gmail.com

zahedy2006
29-02-2008, 10:21
تو محور مختصات سه بعدی سه نقطه روی هر کدوم از محور های x y z داریم که مختصات اونها رو داریم یه نقطه هم تو فضا وجود داره که فاصله اون رو تا سه نقطه ای که روی محور ها هستند داریم و اینجا مختصات نقطه ای که تو فضاست مجهوله چطوری می تونیم مختصات x y z اون رو پیدا کنیم.
هر کی بلده زود جواب ما رو بده چون خیلی لازمش دارم.
اگر می تونید یه فرمول کلی برای این مسئله پیدا کنید که بشه با اون مختصات نقطه مجهول رو بدست آورد لطفا جواب اون رو در صورت امکان به این آدرس ارسال کنید:
khatarat31.5@gmail.com


یه راه همین جوری به ذهنم می رسه (شاید هم کلی نباشه)
مختصات سه نقطه شما که روی محور هستند به صورت های زیر است
x,0,0 - 0,y,0 - 0,0,Z
معلوم هم هست. چون روی محورها هستند
حالا شما مختصات نقطه را با استفاده از فاصله می خواهید.
فاصله هم که به راحتی از فرمول فیثا غورث به دست می آید (توجه کنید که برای هر فاصله یک بار این فرمول را بنویسید چون اگر در حالت کلی بنویسید به یک معادله سه مجهولی خواهید رسید.

از هر بار نوشتن یکی از مختصات ها به دست می آید

mofidy1
29-02-2008, 13:05
با سلام

این سوال از تاپیک مسائل هفته به اینجا منتقل شد.


با سلام کسی هست این سوال را حل کنه ؟

مثلث ABC را که دارای زاوایای حاده است در نظر می گیریم نیمساز داخلی زاویه اش را رسم می کنیم تا ضلع BC را در نقطه ی L و دایره ی محیطی مثلث را در نقطه ی N قطع کند . از L عمود هایی بر اضلاع AB و AC رسم می کنیم و پای دو عمود را K و M می نامیم . ثابت کنید مساحت چهارضلعی AKNM با مساحت مثلث ABC برابر است.

10 اسفند 1386

mofidy1
29-02-2008, 13:10
با سلام

این سوال نیز از تاپیک مسائل هفته به اینجا منتقل شد.


سلام سواله من نسبتا آسونه می خواستم بدونم جوابه این چی میشه در موردش با دبیرمون به مشکل خوردم.
[ برای مشاهده لینک ، لطفا با نام کاربری خود وارد شوید یا ثبت نام کنید ]
خواهشا کمکم کنید چون به مشکل برخوردم
مرسی امین

10 اسفند 1386

khatarat
29-02-2008, 14:29
تو محور مختصات سه بعدی سه نقطه روی هر کدوم از محور های x y z داریم که مختصات اونها رو داریم یه نقطه هم تو فضا وجود داره که فاصله اون رو تا سه نقطه ای که روی محور ها هستند داریم و اینجا مختصات نقطه ای که تو فضاست مجهوله چطوری می تونیم مختصات x y z اون رو پیدا کنیم.
هر کی بلده زود جواب ما رو بده چون خیلی لازمش دارم.
اگر می تونید یه فرمول کلی برای این مسئله پیدا کنید که بشه با اون مختصات نقطه مجهول رو بدست آورد لطفا جواب اون رو در صورت امکان به این آدرس ارسال کنید:
khatarat31.5@gmail.com


این مسئله احتمالا باید با فرمول های دایره حل بشه ولی وقتی من می خوام از اون استفاده کنم یه معادله بدست می یاد که نمی تونم حلش کنم.

sherlockholmz
04-03-2008, 10:49
تو محور مختصات سه بعدی سه نقطه روی هر کدوم از محور های x y z داریم که مختصات اونها رو داریم یه نقطه هم تو فضا وجود داره که فاصله اون رو تا سه نقطه ای که روی محور ها هستند داریم و اینجا مختصات نقطه ای که تو فضاست مجهوله چطوری می تونیم مختصات x y z اون رو پیدا کنیم.
هر کی بلده زود جواب ما رو بده چون خیلی لازمش دارم.
اگر می تونید یه فرمول کلی برای این مسئله پیدا کنید که بشه با اون مختصات نقطه مجهول رو بدست آورد لطفا جواب اون رو در صورت امکان به این آدرس ارسال کنید:
khatarat31.5@gmail.com

سلام
[ برای مشاهده لینک ، لطفا با نام کاربری خود وارد شوید یا ثبت نام کنید ]


با توجه به دلتا،ميتوان جوابهاي T را بدست آوردو پس از آن با قرار دادن در فرمولهاي 2و3و4 ،x,y,z يعني مختصات نقطه خواسته شده بدست مي آيد،مثلا" اگر 'T=T باشدداريم:


[ برای مشاهده لینک ، لطفا با نام کاربری خود وارد شوید یا ثبت نام کنید ]


موفق باشيد.

baghali91
05-03-2008, 17:14
دوستان سلام.من رشته ریاضی سال دوم هستم.قرار کنفرانس در مورد توابع مثلثاتی معکوس(آرک ها) سر کلاس بدم.میخواستم اگه میشه کمکم کنید و اگه مطلبی هست که میتونه کمک کنه اینجا بذارید.در ضمن در سطح سال دوم باشه.خیلی ممنون

khatarat
05-03-2008, 21:06
سلام
[ برای مشاهده لینک ، لطفا با نام کاربری خود وارد شوید یا ثبت نام کنید ]


با توجه به دلتا،ميتوان جوابهاي T را بدست آوردو پس از آن با قرار دادن در فرمولهاي 2و3و4 ،x,y,z يعني مختصات نقطه خواسته شده بدست مي آيد،مثلا" اگر 'T=T باشدداريم:


[ برای مشاهده لینک ، لطفا با نام کاربری خود وارد شوید یا ثبت نام کنید ]


موفق باشيد.



تشکر بسیار زیاد از شما بخاطر این جواب کاملی که دادید واقعا خوشحالمون کرد چون خیلی وقت بود که دنبال جوابش می گشتم.
فقط من یکم قاتی کردم اگه می شه یه مثال عددی ازش حل کنید ببینم چطوری باید ازش استفاده کرد.ممنون می شم.

khatarat
06-03-2008, 10:32
استاد اگه یه id چیزی بدید ممنون می شم تا چند تا سوال داشتم ازتون بپرسم.

baghali91
06-03-2008, 13:10
دوستان سلام.من رشته ریاضی سال دوم هستم.قرار کنفرانس در مورد توابع مثلثاتی معکوس(آرک ها) سر کلاس بدم.میخواستم اگه میشه کمکم کنید و اگه مطلبی هست که میتونه کمک کنه اینجا بذارید.در ضمن در سطح سال دوم باشه.خیلی ممنون

کسی ما رو تخویل نمیگیره؟!!

khatarat
06-03-2008, 13:46
یه نقطه داریم که می دونیم مختصات اون تو دستگاه دو بعدی چیه می خوایم این نقطه رو حول نقطه ای دیگر که مختصات اون رو هم می دونیم بچرخانیم و سپس مختصات جدید نقطه چرخانده شده رو بدست بیاریم.

khatarat
06-03-2008, 13:47
یه نقطه داریم که می دونیم مختصات اون تو دستگاه دو بعدی چیه می خوایم این نقطه رو حول نقطه ای دیگر که مختصات اون رو هم می دونیم بچرخانیم و سپس مختصات جدید نقطه چرخانده شده رو بدست بیاریم
id:khatarat31_5

mohammad96
07-03-2008, 18:16
یه نقطه داریم که می دونیم مختصات اون تو دستگاه دو بعدی چیه می خوایم این نقطه رو حول نقطه ای دیگر که مختصات اون رو هم می دونیم بچرخانیم و سپس مختصات جدید نقطه چرخانده شده رو بدست بیاریم
id:khatarat31_5

سلام!

[ برای مشاهده لینک ، لطفا با نام کاربری خود وارد شوید یا ثبت نام کنید ]

khatarat
08-03-2008, 09:34
سلام
[ برای مشاهده لینک ، لطفا با نام کاربری خود وارد شوید یا ثبت نام کنید ]


با توجه به دلتا،ميتوان جوابهاي T را بدست آوردو پس از آن با قرار دادن در فرمولهاي 2و3و4 ،x,y,z يعني مختصات نقطه خواسته شده بدست مي آيد،مثلا" اگر 'T=T باشدداريم:


[ برای مشاهده لینک ، لطفا با نام کاربری خود وارد شوید یا ثبت نام کنید ]



موفق باشيد.




آقا کسی نیست این فرمول رو کمی برا ما توضیح بده یا یه مثال عددی ازش حل کنه؟
خیلی لازم دارم

khatarat
09-03-2008, 07:28
آقا کسی نبود به ما کمک کنه خیلی لازمش دارم

mohammad96
10-03-2008, 02:55
آقا کسی نبود به ما کمک کنه خیلی لازمش دارمبا سلام!مقادیر x1,y1,z1 که معلومه، (A(x,y,z هم باید بدست بیاریم و a ,b ,c هم فاصله نقطه A تا محورهای x,y,z است. در مرحله بعد هم سه معادله سه مجهولی بدست میاد که باید حلش کرد. اگه یه برنامه کامپیوتری هم مانند MATLAB داشته باشیم ، مساله تا سه شماره (شاید هم کمتر ) حل میشه !!! دیگه چی از این بهتر ؟؟!!

khatarat
12-03-2008, 09:53
سلام
[ برای مشاهده لینک ، لطفا با نام کاربری خود وارد شوید یا ثبت نام کنید ]


با توجه به دلتا،ميتوان جوابهاي T را بدست آوردو پس از آن با قرار دادن در فرمولهاي 2و3و4 ،x,y,z يعني مختصات نقطه خواسته شده بدست مي آيد،مثلا" اگر 'T=T باشدداريم:


[ برای مشاهده لینک ، لطفا با نام کاربری خود وارد شوید یا ثبت نام کنید ]


موفق باشيد.



من این روش رو امتحان کردم جواب نداد البته من نفهمیدم این n ها از کجا اومدن خودم بدست اوردم جواب اشتباه شد حالا یکی توضیح بده یا اینکه اون سه معادله سه مجهول اول رو توضیح بده چطوری می شه حل کرد من هر کاری کردم نتونستم حل کنم
نیاز خیلی خیلی فوری دارم

khatarat
12-03-2008, 09:58
با سلام!مقادیر x1,y1,z1 که معلومه، (A(x,y,z هم باید بدست بیاریم و a ,b ,c هم فاصله نقطه A تا محورهای x,y,z است. در مرحله بعد هم سه معادله سه مجهولی بدست میاد که باید حلش کرد. اگه یه برنامه کامپیوتری هم مانند MATLAB داشته باشیم ، مساله تا سه شماره (شاید هم کمتر ) حل میشه !!! دیگه چی از این بهتر ؟؟!!

استاد من که نتونستم این سه معادله سه مجهول رو حل کنم اگه شما می تونید لطفا برام حل کنید با تشکر

mohammad96
12-03-2008, 23:19
استاد من که نتونستم این سه معادله سه مجهول رو حل کنم اگه شما می تونید لطفا برام حل کنید با تشکر

عدد ها رو بديد ، منم جوابشو ميدم. :46:

Blossom
13-03-2008, 09:46
Salam. Aval mazerat ke en type mikonam. Ba mobilam vasl shodam ke farsi nadare. Ye antegral daram ke mamnoun misham komakam konid ta halesh konam:

برای مشاهده محتوا ، لطفا وارد شوید یا ثبت نام کنید
in alamat ^ ro alamate tavan dar nazar begirin.

saeed666
13-03-2008, 12:08
سلام...

من 1 سوال معادلات دیفرانسیل داشتم . ممنونم میشم كسی بتونه برام حلش كنه .

سوال : حل معادله دیفرانسیل [ برای مشاهده لینک ، لطفا با نام کاربری خود وارد شوید یا ثبت نام کنید ] .

اگر كسی تونست حلش كنه لطفا تو پیغام خصوصی بگه .

ممنون...

saeed666
13-03-2008, 13:36
سلام...

ممنونم میشم كسی بتونه برام حلش كنه . سوال معادلات دیفرانسیله .

سوال : حل معادله دیفرانسیل [ برای مشاهده لینک ، لطفا با نام کاربری خود وارد شوید یا ثبت نام کنید ] .

اگر كسی تونست حلش كنه لطفا تو پیغام خصوصی جوابشو بگه .

ممنون...

mohammad96
13-03-2008, 15:27
Salam. Aval mazerat ke en type mikonam. Ba mobilam vasl shodam ke farsi nadare. Ye antegral daram ke mamnoun misham komakam konid ta halesh konam:

برای مشاهده محتوا ، لطفا وارد شوید یا ثبت نام کنید
in alamat ^ ro alamate tavan dar nazar begirin.


برای مشاهده محتوا ، لطفا وارد شوید یا ثبت نام کنید

Blossom
13-03-2008, 19:47
برای مشاهده محتوا ، لطفا وارد شوید یا ثبت نام کنید

mishe begin oun Atg az koja oumad? Yani che rabete e estefade kardin?
Rasti man natounestam parametr atan ro dark konam, mishe vazeh tar benvisinesh?

mohammad96
13-03-2008, 23:16
mishe begin oun Atg az koja oumad? Yani che rabete e estefade kardin?
Rasti man natounestam parametr atan ro dark konam, mishe vazeh tar benvisinesh?

البته منظور از log همون Ln بود و ديگه اينكه اينو نرم افزار matlab حل كرده :46:

[ برای مشاهده لینک ، لطفا با نام کاربری خود وارد شوید یا ثبت نام کنید ]

saeed666
13-03-2008, 23:48
سلام...

من با مطلب کار نکردم ولی ببین این سوال مار و هم میتونه حل کنه ؟

ممنون...

Blossom
14-03-2008, 00:59
البته منظور از log همون Ln بود و ديگه اينكه اينو نرم افزار matlab حل كرده :46:

[ برای مشاهده لینک ، لطفا با نام کاربری خود وارد شوید یا ثبت نام کنید ]

aagha tashakor. Albate engar shoma tabe ro eshtebah besh dadi. Bayad ln(1-x) ro jaygozine ln(x-1) koni.
Oun arc tan ham az in rabete be dast oumade:
integral du/(a^2 + u^2) = (atan(u/a))/a+C
albate ^ ro tavan farz konid!
Az in ke begzarim, engar in matlab jaleb tar az ounie ke fekr mikardam. Bayad giresh biaram!

mohammad96
14-03-2008, 01:26
aagha tashakor. Albate engar shoma tabe ro eshtebah besh dadi. Bayad ln(1-x) ro jaygozine ln(x-1) koni.
Oun arc tan ham az in rabete be dast oumade:
integral du/(a^2 + u^2) = (atan(u/a))/a+C
albate ^ ro tavan farz konid!
Az in ke begzarim, engar in matlab jaleb tar az ounie ke fekr mikardam. Bayad giresh biaram!

سلام!
آره با شما درباره matlab موافقم (البته كارهاي اينچنيني رو بكمك kernel برنامه Maple انجام ميده). اينكه فرموديد( ln(1-x ، فكر نكنم چون همون اوليه جواب ميده!

ممنون

Blossom
14-03-2008, 01:48
Ajibe! Man hesab mikonam ln(1-x) dar miad.
1-x^3=(1-x)(1+x+x^2)
chon ina to makhraj hastan ba ye tajzie ye kasr miresam be:
(1/3) (integral (du/(1-u))+integral ((u+2)du/(u^2+u+1)))
ke midounim :
integral (du/(1-u)) mishe -ln(1-u)
gheir az ine?

trainercodes
14-03-2008, 12:28
اقا یکی این رو حل کنه:
تعداد جواب های این معادله در مجموعه ی اعداد طبیعی چند تا هست(مجهول ها X,y,zهستند یعنی جواب های این معادله باید برای تمام مقادیر Nصدق کنند)
(x^6n-1) + (y^6n) = ( Z^6n+1 )
تذکر:^نشانه ی توان است یعنی مثلا X^6n-1 معادل X به توان 6n-1است

mohammad96
14-03-2008, 19:11
Ajibe! Man hesab mikonam ln(1-x) dar miad.
1-x^3=(1-x)(1+x+x^2)
chon ina to makhraj hastan ba ye tajzie ye kasr miresam be:
(1/3) (integral (du/(1-u))+integral ((u+2)du/(u^2+u+1)))
ke midounim :
integral (du/(1-u)) mishe -ln(1-u)
gheir az ine?

سلام!
فکر می کنم هر دو جواب درست باشه! اشکالی هم که نداره . روش حل برنامه این طور بوده که ابتدا صورت و مخرج صورت مساله رو در منفی یک ضرب میکنه و .... .

mohammad96
14-03-2008, 19:15
سلام...

من با مطلب کار نکردم ولی ببین این سوال مار و هم میتونه حل کنه ؟

ممنون...

سلام!
نه matlab تونست حلش کنه نه maple ؟؟!!! maple میگه از نوع Abel-second kind-class A ولی ... حلش نمیکنه.

Blossom
14-03-2008, 22:24
سلام!
فکر می کنم هر دو جواب درست باشه! اشکالی هم که نداره . روش حل برنامه این طور بوده که ابتدا صورت و مخرج صورت مساله رو در منفی یک ضرب میکنه و .... .

albate age az ln(1-u) va ln(u-1) moshtagh begirim, ye manfi fargh mikone. Shoma ye bar dige test konid. Albte shayad man eshtebah mikonam.

mohammad96
15-03-2008, 16:25
albate age az ln(1-u) va ln(u-1) moshtagh begirim, ye manfi fargh mikone. Shoma ye bar dige test konid. Albte shayad man eshtebah mikonam.
سلام!
امتحان كردم و براي دو حالت جواب يكي درمياد!

cheeky
15-03-2008, 19:14
سلام
ببخشید سوالم خیلی مبتدیه.
مشتق یعنی چه؟خواهشا مفهومی توضیح بدید.مثلا مشتق x^2 میشه 2x بعد میشه 2 بعد میشه صفر.اینا اصلا چه رابطه ای با همدارن؟در واقع سوالم اینه که مشتق یعنی چی؟

zahedy2006
15-03-2008, 21:11
سلام
ببخشید سوالم خیلی مبتدیه.
مشتق یعنی چه؟خواهشا مفهومی توضیح بدید.مثلا مشتق x^2 میشه 2x بعد میشه 2 بعد میشه صفر.اینا اصلا چه رابطه ای با همدارن؟در واقع سوالم اینه که مشتق یعنی چی؟


مشتق از لحاظ کاربردی یعنی شیب نمودار در نقطه ای خاص و البته کاربرد های دیگر نظیر ماکزیمم و ... .
در مثال شما اگر در نقطه ای به طول x در نمودار x^2 خط مماسی رسم کنید شیب آن 2x خواهد بود.

اگر کاملا مفهومی می خواهید به کتاب حسابان مراجعه کنید. (به عنوان مرجع خوب و فارسی)

cheeky
15-03-2008, 21:56
مشتق از لحاظ کاربردی یعنی شیب نمودار در نقطه ای خاص و البته کاربرد های دیگر نظیر ماکزیمم و ... .
در مثال شما اگر در نقطه ای به طول x در نمودار x^2 خط مماسی رسم کنید شیب آن 2x خواهد بود.

اگر کاملا مفهومی می خواهید به کتاب حسابان مراجعه کنید. (به عنوان مرجع خوب و فارسی)

مرسی.خیلی کمک کردی.

mohammad96
20-03-2008, 02:07
با سلام!
يه سوالي برام پيش اومده و اون اينه كه معادله پارامتري دو خم در فضاي سه بعدي رو داريم. اين دوتا خم هم همديگر رو در يك نقطه قطع مي كنن. حالا براي پيدا كردن اين نقطه ما سه معادله و دو مجهول خواهيم داشت؟ آيا درسته ؟ اگه نه راهش چيه؟

ممنونم!